PrepU Chapter 40-41, 36-38

अब Quizwiz के साथ अपने होमवर्क और परीक्षाओं को एस करें!

The gastrointestinal laboratory nurse is learning about small intestine secretions. Which explanation is most accurate?

"An extensive array of mucus-producing glands, called Brunner glands, is concentrated where contents from the stomach and secretions from the liver and pancreas enter the duodenum." Explanation: An extensive array of mucus producing glands, called Brunner glands, is concentrated at the site where the contents of the stomach and secretions from the liver and pancreas enter the duodenum. These glands secrete large amounts of alkaline mucus. Sympathetic stimulation causes a decrease in mucus production, leaving this area more susceptible to irradiation. As a result, ulcers are four times more likely to occur in the duodenum than in the stomach.

The nurse is caring for a female client with cholelithiasis. When teaching the client about the disease, the nurse includes which of these points?

"Gallstones have developed, which are typically composed of cholesterol." Explanation: Cholelithiasis or gallstones is caused by precipitation of substances contained in bile, mainly cholesterol and bilirubin. It is most common in women, multiple pregnancies, those taking oral contraceptives or those who are obese.

A middle-aged adult is diagnosed with diverticular disease based on recent history and the results of a computed tomography (CT) scan. Which statement demonstrates an accurate understanding of this diagnosis?

"I suppose I should try to eat a bit more fiber in my diet." Explanation: Increased bulk/fiber is important in both the prevention and treatment of diverticular disease. Overuse of laxatives is not linked to diverticular disease, and heartburn and indigestion are not specific signs of the problem. An organic diet and the use of dietary supplements are not key treatments.

A client experiences an increase in cortisol as a result of Cushing disease. Which hormonal responses demonstrate the negative feedback mechanism?

Decreased adrenocorticotropic hormone (ACTH) Explanation: Negative feedback occurs when secretion of one hormone causes a reduction in the secretion of the hormone that stimulates production of the first hormone. In this case, ACTH manufactured by the anterior pituitary gland would normally stimulate the release of cortisol, but with the increase of cortisol produced by the secreting tumor, enough cortisol already floods the system that there should be a reduction in the ACTH level.

Which manifestation indicates a client is at risk for developing diabetes mellitus?

2 hour oral GTT 175 mg/dL (9.7 mmol/L) Explanation: Laboratory values that are considered normal are hemoglobin A1C less than 6.5 percent, fasting plasma glucose of (FPG) less than 100 mg/dL or less than 140 mg/dL 2 hours after an oral glucose tolerance test (GTT). A hemoglobin A1C value that is greater than or equal to 6.5 percent; a fasting blood glucose greater than 126 mg/dL, or a blood glucose level greater than 200 mg/dL 2 hours after a glucose tolerance test (GTT) indicate diabetes mellitus. Values between these levels are considered to place clients at increased risk for diabetes mellitus. Potassium levels do not directly correlate with a diagnosis of diabetes mellitus.

Which individual most likely faces the greatest risk of developing Clostridium difficile colitis?

A 79-year-old hospital client who is being treated with broad-spectrum antibiotics Explanation: C. difficile colitis is associated with antibiotic therapy; C. difficile is noninvasive, and development of C. difficile colitis requires disruption of normal intestinal flora. Peptic ulcers, hyperbilirubinemia, and Crohn disease are not common risk factors for the development of C. difficile colitis.

The most common forms of peptic ulcer are duodenal and gastric ulcers. What are the most common risk factors for peptic ulcer disease?

Helicobacter pylori (H. pylori) Explanation: Perforation occurs when an ulcer erodes through all the layers of the stomach or duodenum wall. H. pylori promotes the development of peptic ulcers by inducing inflammation and stimulation of cytokines and other mediators of inflammation that contribute to mucosal damage. There is no convincing evidence that dietary factors play a role in development of peptic ulcers.

The nurse is caring for a client with diabetes. Which of these findings is cause for concern, leading the nurse to initiate client education?

Hemoglobin A1C level is 8.7%. Explanation: Glycated hemoglobin test (hemoglobin A1c) should be 6% to 7% in clients with diabetes. Maintaining the desired level reduces complications of diabetes.

Which statement best explains the function of hormone receptors?

Hormone receptors recognize a specific hormone and translate the hormonal signal into a cellular response. Explanation: Hormone receptors are complex molecular structures that are located either on the surface of or inside target cells. The function of these receptors is to recognize a specific hormone and translate the hormonal signal into a cellular response.

A man is brought into the emergency department by paramedics who state that the client passed out on the street. The man smells of alcohol, and when roused says he has not eaten since yesterday. He is wearing a medic alert bracelet that says he is a diabetic. What would the nurse suspect as a diagnosis?

Hypoglycemia Explanation: Alcohol decreases liver gluconeogenesis, and people with diabetes need to be cautioned about its potential for causing hypoglycemia, especially if alcohol is consumed in large amounts or on an empty stomach.

A nurse is assessing a client who is experiencing hypoglycemia caused by an insulin reaction. The client is conscious and can follow directions. Which intervention is most appropriate at this time?

Immediately administer a rapidly absorbed form of glucose (preferably via oral route if the client is alert enough to swallow). Explanation: The most effective treatment of an insulin reaction is the immediate administration of 15 g of glucose in a concentrated carbohydrate source. Alternative methods for increasing blood glucose may be required when the person having the reaction is unconscious or unable to swallow. Glucagon may be given intramuscularly or subcutaneously. Glucagon acts by hepatic glycogenolysis to raise blood glucose. In situations of severe or life-threatening hypoglycemia, administer glucose (20 to 50 mL of a 50% solution) intravenously.

The nurse is discussing positive feedback mechanisms. Which example best explains this mechanism?

Increased estradiol production causes increased follicle-stimulating hormone (FSH) production. Explanation: The positive feedback mechanism regulates hormones as follows: a rising level of a hormone causes another gland to release a hormone that is stimulating to the first. The other choices represent negative feedback mechanisms of control.

When caring for the client with hepatic failure, the nurse recognizes that which problem places the client at increased risk for bleeding?

Increased prothrombin time Explanation: An increased prothrombin time would increase the risk for bleeding. Another factor that contributes to increased bleeding risk in clients with liver failure is malabsorption of vitamin K (decrease), which further impairs the synthesis of clotting factors. Increased platelets would not cause increased bleeding but rather increase the risk of clotting, while red blood cell count is not relevant.

What is the most common mechanism of hormone control?

Negative feedback Explanation: With negative feedback, the most common mechanism of hormone control, some feature of hormone action directly or indirectly inhibits further hormone secretion so that the hormone level returns to an ideal level or set point.

The swallowing reflex is an ordered sequence of events. In which phase is the epiglottis moved so that it covers the larynx?

Pharyngeal phase Explanation: In the pharyngeal phase, which is involuntary, the vocal cords are pulled together and the epiglottis is moved so that it covers the larynx.

As part of maintaining homeostasis, why are hormones, secreted by endocrine cells, continuously inactivated?

Prevent accumulation Explanation: Continuous inactivation of secreted hormones is necessary to prevent accumulation that could disrupt the feedback mechanism. Increased secretion stimulates production of more receptor sites. Metabolic waste absorption is not a function of the endocrine system.

When teaching a client about chronic pancreatitis, which of these does the nurse relate is the long term result of this condition?

Progressive destruction of the organ Explanation: The chief distinction between acute and chronic pancreatitis is the irreversibility of pancreatic function with chronic pancreatitis. Chronic pancreatitis is characterized by progressive destruction of the exocrine pancreas, by fibrosis, and, in the later stages, by destruction of the endocrine pancreas. While the most common cause of chronic pancreatitis is alcohol abuse, it is not the consequence.

Which manifestation would a nurse expect when assessing a child with insufficient growth hormone (GH) secretion?

Rank below 10% on the growth chart Explanation: GH stimulates growth of bone and muscle and promotes protein synthesis and fat metabolism and decreased carbohydrate metabolism. A child lacking sufficient GH would demonstrate lack of growth but not the mental delays seen in childhood hypothyroidism.

Which carbohydrate is matched to its correct enzyme needed for digestion?

Sucrose and sucrase Explanation: Sucrose is a dietary carbohydrate digested by sucrase, an enzyme. This produces the monosaccharides fructose and glucose.

A client with long-standing type 2 diabetes is surprised to see high blood sugar readings while recovering from an emergency surgery. Which factor may have contributed to the client's inordinately elevated blood glucose levels?

The stress of the event caused the release of adrenal cortical hormones. Explanation: Elevation of glucocorticoid levels (i.e., cortisol), such as during stressful events, can lead to derangements in glucose metabolism. Tissue trauma does not cause gluconeogenesis, and illness does not inhibit the action of glucagon. The dawn phenomenon is not a likely cause of the client's disruption in blood sugar levels.

The nurse is teaching a client who has been newly diagnosed with hypothyroidism about the function of the thyroid. Which statement about the role of the thyroid gland is most accurate?

The thyroid gland is responsible for increasing the metabolic rate. Explanation: The thyroid gland produces thyroid hormones, T3 and T4. These hormones increase the metabolic rate; increase protein and bone turnover; increase responsiveness to catecholamines; are necessary for fetal and infant growth and development. The parathyroid gland regulates calcium metabolism. The adrenal glands regulate "flight or fight" and the testes or ovaries regulate development of secondary sex characteristics.

Which statement is true concerning gastric enterocytes?

They secrete enzymes that aid in digestion of proteins. Explanation: Gastric enterocytes secrete enzymes that aid in the digestion of carbohydrates and proteins.

A rare condition caused by gastrin-secreting tumors most commonly found in the small intestine or pancreas is called:

Zollinger-Ellison syndrome Explanation: Zollinger-Ellison syndrome is a rare condition caused by a gastrin-secreting tumor (gastrinoma).

A client is diagnosed with Addison disease. What statement by the client indicates an understanding of the discharge instructions by the nurse?

"I will have to take my medication for the rest of my life." Explanation: Addison disease, like type I diabetes, is a chronic metabolic disorder that requires lifetime hormone replacement therapy. The daily regulation of the chronic phase of Addison disease is usually accomplished with oral replacement therapy, with higher doses being given during periods of stress.

What is considered the normal amount of serum bilirubin found in the blood?

<1.5 mg/dL Explanation: Usually, only a small amount of bilirubin is found in the blood; the normal level of total serum bilirubin is <1.5 mg/dL.

A student is studying gallbladder function. Which gastrointestinal hormone stimulates contraction of the gallbladder?

Cholecystokinin Explanation: Cholecystokinin, which is secreted in the duodenum and jejunum, stimulates contraction of the gallbladder and secretion of pancreatic enzymes.

Which statement is true concerning the digestion of carbohydrates?

Carbohydrates are broken down into monosaccharides before being absorbed. Explanation: Carbohydrates must be broken down into monosaccharides or simple sugars before they can be absorbed from the small intestine. They are not excreted without being digested nor do they need hydrochloric acid to be absorbed.

The vesicle-mediated pathway has a role in synthesis and release of which hormones? Select all that apply.

Dopamine Follicle-stimulating hormone Antidiuretic Hormone

Cyclic adenosine monophosphate (cAMP) performs which role in the functioning of the endocrine system?

Acting as a second messenger to mediate hormone action on target cells Explanation: cAMP is one of the most common second messengers, whose role is to generate an intracellular signal in response to cell surface receptor activation by a hormone. cAMP does not mediate hormone synthesis, act as a receptor itself, or inactivate hormones.

The nurse is caring for a client with hepatic encephalopathy who is receiving neomycin. Which of these does the nurse monitor to determine if a positive outcome to the medication has resulted?

Ammonia level Explanation: A nonabsorbable antibiotic, such as neomycin, may be given to eradicate bacteria from the bowel and thus prevent ammonia production.

Which complication of acromegaly can be life threatening?

Cardiac structures increase in size Explanation: While all the complications can exist, it is the enlargement of the heart and accelerated atherosclerosis that may lead to an early death. The teeth become splayed, causing a disturbed bite and difficulty in chewing. Vertebral changes often lead to kyphosis, or hunchback. Bone overgrowth often leads to arthralgias and degenerative arthritis of the spine, hips, and knees. Virtually every organ of the body is increased in size.

Serous fluid is contained in the:

Peritoneal cavity Explanation: Between the visceral and parietal peritoneum is the peritoneal cavity. The peritoneal cavity contains fluid secreted by the serous membranes. This serous fluid forms a moist and slippery surface that prevents friction between the continuously moving abdominal structures. The other options do not contain serous fluid.

A 68-year-old man who has smoked for at least 50 years reports that lately he feels as though food is "getting stuck" in his throat. At first this was a problem just with dry food, but now his morning oatmeal is getting stuck. On questioning, he reports drinking at least five alcoholic beverages nearly every day. His problem is most likely:

squamous cell carcinoma of the esophagus. Explanation: Alcohol and tobacco use are the main risk factors for squamous cell carcinoma of the esophagus and dysphagia is a common presenting complaint. Hiatal hernias and GERD do not cause dysphagia. Scleroderma is unrelated to the client's presentation or history.

A nurse is assessing an older adult with reports of constipation, for which the client often takes over-the-counter medications. What assessment should the nurse perform to address the etiology of the client's problem?

Medication regimen for drugs like anticholinergic agents or calcium Explanation: Constipation is attributable to numerous factors, including low fluid intake and medications. Drugs such as opioids, anticholinergic agents, calcium channel blockers, diuretics, calcium, iron supplements and aluminum antacids tend to cause constipation. Diverticula, rectal fissures, and hemorrhoids are consequences rather than causes of constipation. Abdominal surgery rarely causes constipation unless it is comparatively recent.

The nurse is educating a newly diagnosed client with Hashimoto thyroiditis who is to be discharged from the acute care facility. What should the nurse be sure to include in the education to prevent complications?

The client should be informed about the signs and symptoms of severe hypothyroidism and the need for early intervention. Explanation: Prevention is preferable to treatment and entails special attention to high-risk populations, such as women with a history of Hashimoto thyroiditis. These persons should be informed about the signs and symptoms of severe hypothyroidism and the need for early medical treatment.

A hormone has been synthesized in the rough endoplasmic reticulum of an endocrine cell after which it has moved into the Golgi complex, been packaged in a vesicle, and been released into circulation. Which hormone is synthesized and released in this manner?

Insulin Explanation: Insulin is a peptide hormone; as such, its synthesis and release are vesicle mediated. Glucocorticoids (such as cortisol), androgens (such as testosterone), and estrogens are synthesized by non-vesicle-mediated pathways.

Which statement would help a nurse best explain an "incretin effect"?

"It is an increase in insulin release after ingestion of food." Explanation: An incretin effect means that there is an increase in insulin release after an oral glucose load or food ingestion.

A client takes 650 mg of aspirin every 4 hours daily for reports of joint pain. Which statement should be included in the client's teaching plan?

"This medication can damage gastric mucosa." Explanation: Aspirin and nonsteroidal anti-inflammatory drugs can damage the gastric mucosa. Partial thromboplastin is not affected, nor is hemoglobin or hematocrit. The medication should help to decrease inflammation in the joints.

A client has developed a tumor of the posterior pituitary gland. The client is at risk for problems with secretions of:

Antidiuretic hormone (ADH) and oxytocin Explanation: The posterior pituitary secretes ADH and oxytocin/vasopressin, while the anterior pituitary secretes the hormones listed in the other choices.

A 16-year-old adolescent is hospitalized after ingesting 20 g of acetaminophen in a suicide attempt. The care team would recognize that this client faces risk for which potential complication?

Toxic hepatitis Explanation: Among the manifestations of the direct, predictable liver injuries that accompany overdoses of acetaminophen is toxic hepatitis with tissue necrosis. HDV infection and secondary biliary cirrhosis do not typically result from hepatotoxic drugs. While portal hypertension may develop, it would result from the toxicity.

A nurse who works in the office of an endocrinologist is orienting a new staff member. Which teaching point should the nurse include in the orientation?

"A single hormone can act on not only one process or organ but often on several different locations or processes." Explanation: A single hormone can exert various effects in different tissues, or conversely, a single function can be regulated by several different hormones. Hormones act both distantly from their source and more locally, as in the case of autocrine and paracrine actions. Hormones are normally present at all times.

A client with a history of cancer that metastasized to the liver has arrived at the outpatient clinic to have a paracentesis performed. The physician anticipates that the client will have more than 5 L of fluid removed. The physician has prescribed intravenous albumin following the procedure. The client asks why she needs "more fluids in my vein." The nurse responds:

"Albumin is a volume expander. Since a lot of fluid was removed, you have a decrease in your vascular volume, so without this albumin, your kidneys will try to reabsorb and hold onto water." Explanation: Large-volume paracentesis (removal of 5 L or more of ascitic fluid) may be done in persons with massive ascites and pulmonary compromise. Because the removal of fluid produces a decrease in vascular volume along with increased plasma renin activity and aldosterone-mediated sodium and water reabsorption by the kidneys, a volume expander such as albumin usually is administered to maintain the effective circulating volume.

A client with diabetes mellitus states, "I can eat as many carbohydrates as I want and it will not affect my blood sugar." What is the nurse's best response?

"Carbohydrates are broken down into monosaccharides before being absorbed." Explanation: The client with diabetes mellitus should be taught that carbohydrates are broken down into monosaccharides before being absorbed.

A nurse educator is conducting a course for newly diagnosed diabetes clients. Which statement by a participant should the nurse follow up first?

"I've had a little sore on the sole of my foot for a few days, but I'm sure it will eventually heal." Explanation: Foot problems have been reported as the most common complication leading to hospitalization among people with diabetes. In people with diabetes, lesions of the feet represent the effects of neuropathy and vascular insufficiency and are a clinical priority. There is no particular risk associated with measuring glucose more often than normal. The client's dietary approach is not problematic. The nurse should facilitate the client's eye examinations but a new foot ulcer is a more pressing issue.

A client being treated for diabetes type 2 with insulin presents to a clinic for routine visit. Blood work reveals a HbA1C of 11.0% (high). Which response by the client may account for this abnormal laboratory result?

"My meter broke so I have not been checking my blood glucose levels for a while." Explanation: Glycosylated hemoglobin is hemoglobin into which glucose has been irreversibly incorporated. Because glucose entry into the red blood cell is not insulin dependent, the rate at which glucose becomes attached to the hemoglobin molecule depends on blood glucose; the level is an index of blood glucose levels over the previous 6 to 12 weeks. If the client with diabetes is not monitoring one's blood glucose, the client could be having more periods of hyperglycemia, yet is not aware of the need for increased insulin dosage.

A student is comparing the two patterns of contractions in the small intestine. Which statement is most accurate?

"Segmentation waves function mainly to mix the chyme with the digestive enzymes from the pancreas." Explanation: There are two patterns of contractions in the small intestine: segmentation and peristaltic contractions. Segmentation waves drive the contents forward and backward. They function mainly to mix the chyme with the digestive enzymes from the pancreas and their frequency increases after a meal. Peristaltic movements begin in the duodenum.

A student is studying the esophagus. Which statement made by this student to a faculty member is a correct one?

"The smooth muscle layers provide the peristaltic movements needed to move food along the length of the esophagus." Explanation: The smooth muscle layers provide the peristaltic movements needed to move food along the length of the esophagus. The pharyngoesophageal sphincter, the upper sphincter, keeps air from entering the esophagus and stomach during breathing. The lower esophageal sphincter passes through an opening, or hiatus, in the diaphragm.

What should the nurse teach a client who is diagnosed with chronic hepatitis B infection about treatment?

"The treatment is effective if your liver enzymes return to normal." Explanation: Chronic hepatitis B infection is treated with diet to avoid stressing the liver (no alcohol; moderate fat intake) and medications. Drugs include interferons and nucleotide analog antiretroviral agents (lamivudine, entecavir, and tenofovir). Liver transplantation is considered for end-stage liver failure, but is not as successful for chronic hepatitis B as for chronic hepatitis C. Steroids are used to treat autoimmune hepatitis.

A young child develops type 1A diabetes. The parents ask, "They tell us this is genetic. Does that mean our other children will get diabetes?" The best response by the health care provider would be:

"This autoimmune disorder causes destruction of the beta cells, placing your children at high risk of developing diabetes." Explanation: Type 1 diabetes is subdivided into two types: type 1A, immune-mediated diabetes, and type 1B, idiopathic diabetes. Type 1A diabetes is characterized by autoimmune destruction of beta cells. The other choices are not absolutely correct. The fact that type 1 diabetes is thought to result from an interaction between genetic and environmental factors led to research into methods directed at prevention and early control of the disease. These methods include the identification of genetically susceptible persons and early intervention in newly diagnosed persons with type 1 diabetes.

Following an oral glucose tolerance, a 36-year-old mother of 4 has been diagnosed with gestational diabetes mellitus (GDM), a problem that was not present in any of her previous pregnancies. What should her primary care provider tell her about this new health problem?

"Your baby could become too large or have low blood sugars if we're not vigilant about controlling your sugars." Explanation: Women with GDM are at higher risk for complications of pregnancy, mortality, and fetal abnormalities. Fetal abnormalities include macrosomia, hypoglycemia, hypocalcemia, polycythemia, and hyperbilirubinemia. GDM often persists as type 2 diabetes after delivery. The baby does not face a significantly higher risk of developing diabetes. Nutrition therapy would precede insulin therapy and GDM involves a pancreatic etiology.

How long is the half-life of the hormone aldosterone, which is only 15% protein bound?

25 minutes Explanation: Aldosterone, which is only 15% protein-bound, has a half-life of only 25 minutes. The higher the percentage of protein binding, the longer the half-life will be.

After receiving change-of-shift report about the following four clients, which client should the nurse assess first?

70-year-old returning from PACU following partial thyroidectomy who is extremely agitated, has an irregular pulse rate of 134, and an elevated temperature of 103.2°F (39.6°C) Explanation: Manipulation of a hyperactive thyroid gland during thyroidectomy can cause thyroid storm. It is manifested by very high fever, extreme cardiovascular effects (tachycardia, HF, angina), and severe CNS effects (agitation, restlessness, and delirium). The 22-year-old has normal sodium levels. The 31-year old has a high blood glucose level but not at a critical level. The medication schedule for the 53-year-old is lower priority. It is always preferred to give medications in timely manner; however, thyroid storms are the priority for this group of clients.

A client is prescribed erythromycin for an infection. What manifestations will the nurse recognize that indicate the onset of drug-induced cholestasis?

Jaundice and pruritus Explanation: Jaundice and itching are the early manifestations of drug-induced cholestasis. Clients generally report no change in feelings of well-being. The symptoms subside when the drug is withdrawn.

Which client should the nurse observe most closely for the signs and symptoms of paralytic ileus?

A client who is first day postoperative following gallbladder surgery Explanation: Paralytic ileus is a significant complication of abdominal surgery. The problem is not associated with the use of antidiarrheal medications, obesity, or irritable bowel syndrome.

Which client would be considered to be exhibiting manifestations of "prediabetes"?

A middle-aged overweight adult with a fasting plasma glucose level of 122 with follow-up OGTT of 189 mg/dL (10.49 mmol/L). Explanation: Persons with IFG (impaired fasting plasma glucose [IFG] defined by an elevated FPG of 100 to 125 mg/dL [5.55 to 6.94 mmol/L]) and/or IGT (impaired glucose tolerance [IGT] plasma glucose levels of 140 to 199 mg/dL [7.77 to 11.04 mmol/L] with an OGTT) are often referred to as having prediabetes, meaning they are at relatively high risk for the future development of diabetes as well as cardiovascular disease.

Which individual likely faces the highest risk of developing chronic pancreatitis?

A woman who has 6 to 8 alcoholic beverages each evening Explanation: By far, the most common cause of chronic pancreatitis is long-term alcohol use disorder. The other cited factors are not noted to contribute significantly to the pathogenesis of chronic pancreatitis.

Irritable bowel syndrome is thought to be present in 10% to 15% of the population in the United States. What is its hallmark symptom?

Abdominal pain relieved by defecation with a change in consistency or frequency of stools Explanation: A hallmark of irritable bowel syndrome is abdominal pain that is relieved by defecation and associated with a change in consistency or frequency of stools. Nausea, altered bowel function, and diarrhea are also symptoms of irritable bowel syndrome but not combined with abdominal pain that is unrelieved by defecation. A bowel impaction is not a symptom of irritable bowel syndrome.

Which symptom is often observed in cases of peritonitis?

Abdominal rigidity Explanation: The abdomen is rigid and becomes boardlike because of reflex muscle guarding. The client typically becomes tachycardic, has increased WBC count, and breathes in a shallow way to avoid movement of the abdomen related to the pain.

What is true regarding the process of absorption? Select all that apply.

Absorption moves nutrients from the gastrointestinal tract to the blood stream. Absorption of some substances involves active transport. Absorption of some substances involves diffusion. Explanation: Absorption is the process of moving nutrients and other materials from the external environment in the lumen of the gastrointestinal tract into the internal environment, such as the blood stream. Absorption is accomplished by active transport and diffusion. The absorptive function of the large intestine focuses mainly on water reabsorption. A number of substances require a specific carrier or transport system. For example, vitamin B12 is not absorbed in the absence of intrinsic factor, which is secreted by the parietal cells of the stomach. Transport of amino acids and glucose occurs mainly in the presence of sodium. Water is absorbed passively along an osmotic gradient.

The nurse is assessing a client with thyrotoxicosis and the nurse is explaining how the thyroid gland is stimulated to release thyroid hormones. The nurse should describe what process?

Action of releasing hormones from hypothalamus Explanation: The synthesis and release of anterior pituitary hormones are largely regulated by the action of releasing or inhibiting hormones from the hypothalamus, which is the coordinating center of the brain for endocrine activity such as thyroid activity. There are no direct innervations for hormone release from the thyroid gland, and homeostatic receptors do not exist. Steady-state release of hormones does not occur.

Absorption is a major function of the GI tract. How is absorption accomplished in the GI tract?

Active transport and diffusion Explanation: Absorption is accomplished by active transport and diffusion.

A 79-year-old woman reports a recent onset of "nearly constant heartburn." During the assessment interview, she states that she has "lots of aches and pains." She states that she is not on any prescription medications but often takes aspirin for pain. The nurse should suspect what diagnosis?

Acute gastritis Explanation: Aspirin is often implicated in cases of acute gastritis. H. pylori, gastric cancer, and staphylococcus infections do not normally cause reflux (heartburn) and are not directly linked to aspirin use.

Given the fact that acute pancreatitis can result in severe, life-threatening complications, the nurse should be assessing the client for which complication?

Acute tubular necrosis Explanation: Complications of acute pancreatitis include the systemic inflammatory response, acute respiratory distress syndrome, acute tubular necrosis, and organ failure. Cerebral hemorrhage, bilateral pneumothorax, and complete heart block are not associated with the complications of acute pancreatitis.

A man with a history of excess alcohol consumption for 25 years is admitted with cirrhotic liver disease. To assess the amount of damage to the liver, which lab tests of hepatobiliary function should be monitored? Select all that apply.

Alanine aminotransferase (ALT) gamma-glutamyl transferase (GGT) Aspartate aminotransferase (AST) Explanation: AST, GGT, and ALT are all measures of liver function. Creatinine is a measure of renal function and troponin is a measure of cardiac damage.

A client with history of alcohol abuse is brought to the emergency department after a weekend of heavy drinking, experiencing right upper quadrant pain, anorexia, nausea, jaundice and ascites. The nurse identifies these as manifestations of what disorder?

Alcoholic hepatitis Explanation: Fatty liver occurs when there is an accumulation of fat in the liver cells. The liver enlarges and becomes yellow. The fatty changes are reversible when alcohol intake stops. Alcoholic hepatitis is the next stage of liver disease after fatty liver. It is common when there is a sudden increase in alcohol intake and has a mortality rate of approximately 34 percent. The liver becomes inflamed and necrosis occurs. If the client survives and continues to use alcohol, alcoholic hepatitis develops into alcoholic cirrhosis. The liver develops fine, uniform nodules on the surface. As the disease progresses, the nodules become larger, and blood flow is obstructed resulting in portal hypertension, extrahepatic portosystemic shunts, and cholestasis. Gallbladder cancer occurs insidiously and has similar signs/symptoms as cholelithiasis.

Select the category of hormones that include norepinephrine and epinephrine.

Amines and amino acids Explanation: Hormones can be divided into three categories: (1) amines and amino acids; (2) peptides, polypeptides, proteins, and glycoproteins; and (3) steroids. The amines include norepinephrine and epinephrine. The second category, the peptides, includes polypeptides, proteins, and glycoproteins. The third category consists of the steroid hormones, which are derivatives of cholesterol.

Hormones can be synthesized by both vesicle-mediated pathways and non-vesicle-mediated pathways. What hormones are synthesized by non-vesicle-mediated pathways?

Androgens and estrogens Explanation: Hormones that are synthesized by non-vesicle-mediated pathways include the glucocorticoids, androgens, estrogens, and mineralocorticoids—all steroids derived from cholesterol.

A nurse is reviewing the admission assessment data of a client diagnosed with acute gastritis. The nurse determines that the condition most likely occurred as a result of:

Arthritis treated with high levels of nonsteroidal anti-inflammatory (NSAIDs) agents Explanation: Acute gastritis is most commonly associated with local irritants such as aspirin or other NSAIDs, alcohol, or bacterial toxins. A high-fiber diet and occasional alcohol consumption are not causes. The severe headache may cause stress or nausea but does not cause gastritis.

An older adult client presents with loose mucus-filled stools. The nurse suspects the client has Clostridium difficile. What is a priority assessment for the nurse?

Ask the client about his or her antibiotic use. Explanation: The diagnosis of C. difficile-associated diarrhea requires a careful history, with particular emphasis on antibiotic use.

A client is being screened for colorectal cancer with the fecal occult test. The nurse instructs the client to avoid ingesting which items 3 to 7 days prior to the test? Select all that apply.

Aspirin Citrus fruits Red meats Explanation: Almost all cancers of the colon and rectum bleed intermittently, although the amount of blood is small and usually not apparent in the stools. It therefore is feasible to screen for colorectal cancers using commercially prepared tests for occult blood in the stool. To reduce the likelihood of false-positive test results, people are instructed to avoid NSAIDs such as ibuprofen and aspirin for 7 days before testing; to avoid vitamin C in excess of 250 mg from either supplements or citrus fruits for 3 days before testing; and to avoid red meats for 3 days before testing.

An older adult in a rehabilitation unit is at risk for constipation. Which interventions would be appropriate for the nurse to take? Select all that apply.

Assess the client's current medication regimen Ensure that the client's fluid intake is sufficient Encourage the client to attempt a bowel movement after a meal Explanation: Medications can profoundly affect bowel motility and should be assessed for this effect. Adequate fluid intake is important and bowel movements should be attempted after eating. Scheduling physical activity after a meal is not an action that is commonly used to promote bowel motility. Enemas should not be used as a preventive measure.

When educating a client about type 1 diabetes, the nurse will mention that this type is caused by which mechanism?

Autoimmune destruction of pancreatic beta cells Explanation: Type 1 diabetes involves autoimmune destruction of beta cells. The pathophysiology of type 2 diabetes has both genetic and acquired factors.

A 40-year-old client tells the nurse that a parent died of gastric cancer and that the client wants to do everything one can do to avoid the disease. Which recommendation should the nurse provide?

Avoid smoked and preserved foods. Explanation: Research has demonstrated a correlation between genetic factors, consumption of smoked and preserved foods, autoimmune gastritis, and gastric adenomas or polyps as key risk factors for gastric cancer. Decreased consumption of salted, smoked, and preserved foods in populations has seen a decrease in the incidence of gastric cancer. Increasing fiber intake is beneficial for general health but does not specifically protect against gastric cancer. A low-carbohydrate diet does not confer protection. There has been no widely accepted scientific evidence linking artificial sweeteners to gastric cancer.

A client is experiencing hepatorenal syndrome. Which manifestations will the nurse assess in this client? Select all that apply.

Azotemia Increased serum creatinine Oliguria Ascites Explanation: The hepatorenal syndrome refers to a functional renal failure sometimes seen during the terminal stages of liver failure. It is characterized by progressive azotemia, increased serum creatinine levels, and oliguria. With renal failure and liver failure combined, the development of hepatic encephalopathy occurs; this is characterized by disturbances ranging from a lack of mental alertness to confusion, coma, and convulsion.

The dietitian explains to the client that food is digested and absorbed in the:

Jejunum Explanation: Food is digested and absorbed in the jejunum and ileum. The stomach serves as a food reservoir during the early stages of digestion. The cecum and colon are parts of the large intestine.

Symptoms of gastric cancer include vague epigastric pain, which makes early detection difficult. The nurse would expect a client to undergo which diagnostic examination to determine the location of the gastric cancer?

Barium x-ray Explanation: Diagnosis of gastric cancer is accomplished by a variety of techniques, including barium x-ray studies, endoscopic studies with biopsy, and cytologic studies of gastric secretions. Cytologic studies can prove particularly useful as routine screening tests for persons with atrophic gastritis or gastric polyps. Computed tomography and endoscopic ultrasonography often are used to delineate the spread of a diagnosed stomach cancer. Treatment of choice, depending on location and extent, are surgery, irradiation, and chemotherapy.

A client is to have a serum thyroxine and thyroid stimulating laboratory test performed to assess the baseline status of the hypothalamic-pituitary target cell hormones. When educating the client about the laboratory tests, when would the nurse inform him the test should be obtained?

Before 0800 Explanation: The assessment of hypothalamic-pituitary function has been made possible by many newly developed imaging and radioimmunoassay methods. Assessment of the baseline status of the hypothalamic-pituitary target cell hormones involves measuring the following laboratory specimens (ideally obtained before 0800): serum cortisol, serum prolactin, serum thyroxine and TSH, serum testosterone and estrogen and serum LH/FSH, serum GH, and plasma and urine osmolality.

The results of a 44-year-old obese man's recent diagnostic workup have culminated in a new diagnosis of type 2 diabetes. Which pathophysiologic process underlies the client's new diagnosis?

Beta cell exhaustion due to long-standing insulin resistance Explanation: Exhaustion of the beta cells arising from insulin resistance is characteristic of type 2 diabetes. Beta cell destruction in the absence of an autoimmune reaction is associated with type 1b diabetes, while autoimmune processes contribute to type 1a diabetes.

The client has right upper quadrant pain caused by acute choledocholithiasis. The health care provider suspects the common bile duct is obstructed, based on which flowing lab value?

Bilirubin 15 mg/dL (256.56 µmol/L) (high) Explanation: Choledocholithiasis, stones in the common duct, usually originate in the gallbladder but can form spontaneously in the common duct. Bilirubinuria and an elevated serum bilirubin are present if the common duct is obstructed. With acute cholecystitis, approximately 75% of clients have vomiting. Ascites is common with late-stage liver failure rather than duct obstructions. Bleeding is associated with liver failure due to deficiency of clotting factors and acute pancreatitis due to activated enzymes, causing fat necrosis and hemorrhage from the necrotic vessels.

When caring for the client with diabetic ketoacidosis, the nurse recognizes that fatty acids and ketones may be used for energy by most organs. Which organ does the nurse recognize is reliant on glucose as the major energy source?

Brain Explanation: Although many tissues and organ systems are able to use other forms of fuel, such as fatty acids and ketones, the brain and nervous system rely almost exclusively on glucose as a fuel source. Because the brain can neither synthesize nor store more than a few minutes' supply of glucose, normal cerebral function requires a continuous supply from the circulation.

A hospitalized client with diabetes mellitus has sudden onset of slurred speech; lack of coordination; and cool, clammy skin. What will the nurse do first?

Check blood glucose. Explanation: Hypoglycemia is a medical emergency. Because other conditions can also cause these symptoms, the blood glucose level should be checked first. If in the community where a glucometer may not be readily accessible, it can be appropriate to administer glucose based on symptoms alone. However, in the hospital setting, the nurse should assess the blood glucose first. Then glucose should be given as needed. The client should follow this with a protein and complex carbohydrate to avoid a recurrence of hypoglycemia. Blood glucose should be tested about 15 minutes after the glucose is given to monitor progress.

Crohn disease has a distinguishing pattern in the gastrointestinal (GI) tract. The surface has granulomatous lesions surrounded by normal-appearing mucosal tissue. A complication of the pattern includes:

Fistula formation Explanation: In Crohn disease all layers of the bowel are involved. Complications of Crohn disease include fistula formation, abdominal abscess formation, and intestinal obstruction. Fistulas are tubelike passages that form connections between different sites in the GI tract.

A child is experiencing frequent vomiting over the past 24 hours. What site in the neurologic system is responsible for vomiting?

Chemoreceptor trigger zone Explanation: The act of vomiting is integrated in the vomiting center, which is located in the dorsal portion of the reticular formation of the medulla near the sensory nuclei of the vagus. The vomiting center can be activated directly by irritants or indirectly following input from four different sources, one of which is the chemoreceptor trigger zone (which is activated by chemical agents such as drugs and toxins).

All diseases have risk factors. What is the most significant environmental risk factor for pancreatic cancer?

Cigarette smoking Explanation: In pancreatic cancer, the most significant and reproducible environmental risk factor is cigarette smoking.

The nurse is planning to collect a 24-hour urine sample for hormone assay. In which situation does the nurse collaborate with the health care provider to find an alternate type of testing?

Client has anuria. Explanation: The advantages of a urine test include the relative ease of obtaining urine samples and the fact that blood sampling is not required. The disadvantage is that reliably timed urine collections often are difficult to obtain and rely on adequate renal function. Anuria refers to the absence of urine output.

When assessing the nutritional intake of a client with generalized pancreatic cancer, which statement by the client is consistent with the disease and food intake?

Client states pain becomes worse with food intake. Explanation: Cancer of the body of the pancreas usually impinges on the celiac ganglion, causing pain. The pain usually worsens with ingestion of food or assumption of the supine position.

The incidence of stomach cancer has significantly decreased in the United States, yet it remains the leading cause of death worldwide. The nurse understands the reason for the high mortality rate in stomach cancer is because of which reason?

Clients have few early symptoms of the disease. Explanation: Although the incidence of cancer of the stomach has declined over the past 50 years in the United States, it remains the leading cause of death worldwide. Because there are few early symptoms with this form of cancer, the disease is often far advanced at the time of diagnosis. Diagnosis of gastric cancer is accomplished by a variety of techniques, including barium x-ray studies, endoscopic studies with biopsy, and cytologic studies (e.g., Papanicolaou smear) of gastric secretions. Chronic infection with Helicobacter pylori appears to serve as a cofactor in some types of gastric carcinomas. The bacterial infection causes gastritis, followed by atrophy, intestinal metaplasia, and carcinoma.

A client with hypothyroidism has not taken medication for several months, informing the nurse that she lost her insurance and is unable to afford the medication. When assessing the client's temperature tolerance and skin, what does the nurse anticipate finding? Select all that apply.

Coarse and dry skin and hair Intolerance to cold Decreased sweating Explanation: The client with hypothyroidism experiences an intolerance to cold, decreased sweating, and coarse and dry skin and hair, related to the decrease in metabolic rate from the deficient thyroid secretion.

An older adult client has been placed on a broad-spectrum antibiotic for a recurrent urinary tract infection. Which potential problem would the nurse anticipate in this client?

Colonization of Clostridium difficile Explanation: Clostridium difficile colitis is associated with antibiotic therapy. Almost any antibiotic may cause C. difficile colitis, but broad-spectrum antibiotics with activity against Gram-negative enteric bacteria are the most frequent agents. After antibiotic therapy has made the bowel susceptible to infection, colonization by C. difficile occurs by the oral-fecal route.

Which criterion about insulin would prompt a diagnosis of type 1 diabetes?

Complete failure of insulin secretion Explanation: In type 1 diabetes there is an absolute lack of insulin due to complete failure of the pancreas. In type 2 diabetes some insulin is produced but may not be properly used.

A health care provider suspects a client has developed diverticular disease. Which diagnostic test is usually prescribed to confirm the diagnosis?

Computed tomography (CT) scan Explanation: Diverticular disease may be determined by CT scan or ultrasound. CT scans are the safest and most cost-effective. Due to the risk of peritonitis, barium enema studies should be avoided, and flat-plate radiographs may be used to detect complications. A PET scan measures important body functions, such as blood flow, oxygen use, and sugar (glucose) metabolism.

An older adult client expresses concern about straining to pass stools that are small and hard. What term will the health care provider use to document this in this client's chart?

Constipation Explanation: Constipation can be defined as the infrequent, incomplete, or difficult passage of stools. Diverticula are small pouches that bulge outward through the colon, or large intestine. Irritable bowel syndrome is a group of symptoms that occur together, including repeated pain in the abdomen and changes in bowel movements, which may be diarrhea, constipation, or both. Dehydration occurs when more water and fluids leave the body than enter it. Even low levels of dehydration can be the cause of headaches, lethargy, and constipation.

Colonic microorganisms play a role in the synthesis of which vitamin?

K Explanation: Colonic microorganisms play a role in vitamin synthesis and in absorption of calcium, magnesium, and iron. Colonic flora synthesize vitamin K.

A client has been diagnosed with a defect in the pyloric sphincter. How will the nurse explain the likely consequence of a dysfunctional pyloric sphincter?

Contents from the small intestine may flow back into the stomach. Explanation: The pyloric sphincter is a muscle that serves as a valve controlling stomach emptying and prevents regurgitation of intestinal contents into the stomach. With dysfunction, the client could have reflux from the small intestine back into the stomach but this will not result in slowing motility. The lower esophageal sphincter controls food entering the stomach. Defecation is controlled by the action of the internal and external anal sphincters.

A 24-year-old woman undergoing a screening test is found to have elevated levels of AST, ALT, and IgG, but no antibody-specific markers for viral hepatitis. A liver biopsy reveals inflammation and cellular damage. Which treatment is most likely to be effective for her?

Corticosteroids and immunosuppressant drugs Explanation: This woman's hepatitis is probably caused by an autoimmune disorder rather than a virus. Lamivudine, pegylated interferon/ribavirin, and interferon alfa-2b are all antiviral agents.

A client is diagnosed with adrenocorticotropic hormone deficiency (ACTH) and is to begin replacement therapy. Regarding which type of replacement will the nurse educate the client?

Cortisol replacement therapy Explanation: Cortisol replacement is started when ACTH deficiency is present; thyroid replacement when TSH deficiency is detected; and sex hormone replacement when LH and FSH are deficient. GH replacement is indicated for pediatric GH deficiency, and is increasingly being used to treat GH deficiency in adults.

After having a very stressful day in pathophysiology class, the student knows that which hormone (secreted by the adrenal cortex) will help decrease the effects of stress?

Cortisol, a glucocorticoid Explanation: Glucocorticoids, mainly cortisol, affect metabolism of all nutrients; regulate blood glucose levels; affect growth; have anti-inflammatory action; and decrease effects of stress. The other hormones do not affect stress levels.

The newborn nursery nurse is obtaining a blood sample to determine if a newborn has congenital hypothyroidism. What long-term complication is the nurse aware can occur if this test is not performed and the infant has congenital hypothyroidism?

Cretinism Explanation: Congenital hypothyroidism is a common cause of preventable intellectual disability. It affects approximately 1 in 4000 infants. The manifestations of untreated congenital hypothyroidism are referred to as cretinism. The term does not apply to the normally developing infant in whom replacement thyroid hormones therapy was instituted shortly after birth.

A young client presents reporting diarrhea, fecal urgency, and weight loss. The stool is light-colored and malodorous, and it tends to float and be difficult to flush. The client has also noted tender, red bumps on the shins and reports pain and stiffness in the elbows and knees. Sigmoidoscopy reveals discontinuous, granulomatous lesions; no blood is detected in the stool. Which diagnosis would his care team first suspect?

Crohn disease Explanation: Crohn disease, like ulcerative colitis, causes diarrhea, fecal urgency, weight loss, and systemic symptoms such as erythema nodosum and arthritis. Unlike ulcerative colitis, it also causes steatorrhea but is not as likely to cause blood in the stool. The granulomatous "skip" lesions confirm the diagnosis of Crohn disease. Neither diverticulitis nor colon cancer would cause this combination of symptoms and signs.

Antibody testing has confirmed that a client is positive for hepatitis A virus (HAV). Which statement does the nurse apply as evidence that the client understands the new diagnosis?

I don't know why I didn't bother to get vaccinated against this." Explanation: A vaccine is available for HAV. The disease is normally self-limiting and does not result in carrier status. Transmission is usually by the fecal-oral route rather than sexual transmission.

A nurse is concerned about a client's continual use of nonsteroidal anti-inflammatory drugs (NSAIDs). Which problematic occurrence is the nurse concerned about?

Damage to the gastric mucosa Explanation: One of the important characteristics of the gastric mucosa is resistance to highly acidic secretions that it produces. However, when aspirin, NSAIDs, Helicobacter pylori, ethyl alcohol, or bile salts damage the gastric mucosa, this barrier is disrupted. NSAIDs do not increase the production of gastric acid (proton pump), decrease the production of intrinsic factor, or increase the secretion of gastrin.

A client with type 2 diabetes experiences unexplained elevations of fasting blood glucose in the early morning hours. Which conditions can account for this effect?

Dawn phenomenon Explanation: The dawn phenomenon involves increased levels of fasting blood glucose or insulin requirement during the early morning hours. It is not preceded by hypoglycemia. Circadian release of growth hormone and cortisol may be contributing factors. The other answer selections are not characterized by increased early morning levels of blood glucose.

An adult client presents to the emergency department after an episode of syncope. Blood pressure is low and the pulse is thready and rapid. The client frequently voids large amounts of pale, clear urine. Lung sounds are clear and skin turgor is inelastic. Which endocrine disorder is the likely cause of these manifestations?

Diabetes insipidus Explanation: Normally, antidiuretic hormone (ADH) from the posterior pituitary gland causes the renal tubule to reabsorb water and sodium. When too little ADH is secreted, the client will lose large amounts of fluid and become dehydrated. Despite fluid volume deficit, the urine output will be dilute and in large amounts. In hyperparathyroidism, the client will have an elevated blood calcium level and cardiac dysrhythmias. Fluid volume will remain stable, but the client may develop renal calculi. In myxedema, lack of thyroid hormones causes the client to become sluggish. Urinary output remains normal. Gigantism results from excess production of growth hormone in children and results in large body size. It does not influence urine output.

Which aspect of gastrointestinal function is performed by the brush border enzymes of the villus structures?

Digestion of carbohydrates Explanation: Brush border enzymes aid in the digestion of carbohydrates and proteins. They do not metabolize fat or synthesize vitamin A, and they do not directly contribute to fluid and electrolyte balance.

A client who has been taking acetaminophen 1000 mg every 4 hours presents to the Urgent Care Center with increased abdominal pain, elevated ALT, AST, and bilirubin levels. The nurse suspects the client is experiencing:

Direct hepatotoxic reaction Explanation: Direct hepatotoxic reactions usually are a recognized characteristic of certain drugs. They usually result from drug metabolism and the generation of toxic metabolites from drugs like acetaminophen, isoniazid, and phenytoin. The condition is characterized by elevated ALT, AST, and bilirubin. Idiosyncratic reactions are unpredictable, not related to dose, and sometimes accompanied by features suggesting an allergic reaction. Cholestatic drug reactions result in decreased secretion of bile or obstruction of the biliary tree. The clinical features of chronic hepatitis are extremely variable and not predictive of outcome.

A client newly diagnosed with pancreatic cancer is admitted to begin treatment. Which pain descriptors can be associated with adenocarcinomas of the pancreas?

Dull epigastric pain accompanied by back pain, worse when lying flat and relieved by sitting forward. Explanation: The most common pain with pancreatic cancer is a dull, epigastric pain often accompanied by back pain, often worse in the supine position, and relieved by sitting forward. Sharp, stabbing pain with respirations could be pleurisy among other respiratory problems. Abdominal pain following a meal is usually associated with GERD or gastric ulcers. Cerebral edema causing headaches is not related to pancreatic cancer.

The common bile duct and the pancreatic duct release their products into which section of the intestines?

Duodenum Explanation: The common bile duct and pancreatic duct empty their juices into the duodenum. The antrum is the lower portion of the stomach. Although the ileum and jejunum are also parts of the small intestine, there are no duct openings in those sections.

The common bile duct opens into which part of the gastrointestinal tract?

Duodenum Explanation: The duodenum contains the opening for the common bile duct and the main pancreatic duct.

The nurse suspects that a girl is experiencing precocious puberty based on which physical findings? Select all that apply.

Early breast development Beginning of augmented adrenal androgen production Beginning of menstrual function Explanation: Adolescence begins with development of secondary sex characteristics (11 to 12 years) and ends with cessation of somatic growth (18 to 21 years). This is the period of a major growth spurt, which is more pronounced in boys. The endocrine system is of great importance, with its numerous hormonal changes that cause the initiation and continuation of the growth spurt. Precocious sexual development may be idiopathic or may be caused by gonadal, adrenal, or hypothalamic disease. Diagnosis of precocious puberty is based on physical findings of early thelarche (beginning of breast development), adrenarche (beginning of augmented adrenal androgen production), and menarche (beginning of menstrual function) in girls.

What manifestations will the nurse expect to find when assessing a client with cholestasis? Select all that apply.

Easy bruising Clay-colored stools Pruritus Explanation: The client with acute cholestasis due to extrahepatic obstruction will exhibit manifestations of lack of bile action in the intestines. This includes lack of absorption of fats, leading to a stool that is grayish in color, frothy, and odorous. Lack of bile reduces absorption of vitamins A, D, E, and K so the clients can exhibit changes in vision, lack of bone development, delayed healing, and delayed clotting (which results in bruising easily). Skin and sclera may become icteric (yellowed) and itchy from the deposit of bile salts that would normally be released into the bowel. Cholesterol and cholestasis can be elevated.

A nurse examines the laboratory values of a client in heart failure. Which value indicates a compensatory hormone mechanism?

Elevated atrial natriuretic hormone Explanation: In heart failure, the client experiences fluid backlog in the heart as venous blood continues to return, but cardiac output is reduced. This stretches the atria, which secrete atrial natriuretic hormone (or peptide) to stimulate vasodilation and increased renal excretion of sodium and water. This reduces the volume and the strain in the heart.

A client is admitted with chronic gastritis. The nurse expects which invasive test to be performed to establish the presence of Helicobacter pylori (H. pylori)?

Endoscopic biopsy Explanation: Methods for establishing the presence of H. pylori infection include the carbon urea breath test, the stool antigen test, and endoscopic biopsy for urease testing. The invasive test is the endoscopic biopsy, which removes a tissue sample from the lining of the stomach.

The nurse is assessing a client who has been diagnosed with gastroesophageal reflux disease (GERD). The nurse recognizes which sign/symptom may be associated with GERD? Select all that apply.

Epigastric pain Retrosternal pain Wheezing Hoarseness Explanation: The most frequent symptom of GERD is heartburn. It frequently is severe, occurring 30 to 60 minutes after eating. A recumbent position may increase pain and usually is relieved by sitting upright. Often, the heartburn occurs during the night. Antacids provide prompt, although transient, relief. Other symptoms include belching and chest pain. The pain usually is located in the epigastric or retrosternal area and often radiates to the throat, shoulder, or back. The pain may be confused with angina. The reflux of gastric contents also may produce respiratory symptoms such as wheezing, chronic cough, and hoarseness. Aspirin would increase distress.

Pharmacologic treatment for peptic ulcers has changed over the past several decades. The nurse knows that the goal for pharmacologic treatment is focused on:

Eradicating Helicobacter pylori (H. pylori) Explanation: Treatment of peptic ulcer is aimed at eradicating the cause and promoting a permanent cure for the disease. Pharmacologic treatment focuses on eradicating H. pylori, relieving ulcer symptoms, and healing the ulcer crater. Acid-neutralizing, acid-inhibiting drugs and mucosa-protective agents are used to relieve symptoms and promote healing of the ulcer crater. There is no evidence that special diets are beneficial in treating peptic ulcer.

Which pathophysiologic phenomenon may result in a diagnosis of Cushing disease?

Excess ACTH production by a pituitary tumor Explanation: Three important forms of Cushing syndrome result from excess glucocorticoid production by the body. One is a pituitary form, which results from excessive production of ACTH by a tumor of the pituitary gland. Hypopituitarism and destruction of the adrenal cortex are associated with Addison disease. Disruption of the HPA system is not implicated in the etiology of Cushing disease.

A 15-year-old child with type 1 diabetes asks nurse about the potential to "lose sight." Which response would be the most appropriate?

Explain that many people with diabetes experience some complications like retinopathy, but these are best prevented with tight control of glucose levels. Explanation: The types of microvascular complications that occur in diabetes mellitus can include neuropathy, retinopathy, nephropathy, and disorders of gastrointestinal motility. In the United States, diabetes is a leading cause of vision loss and blindness as well as chronic kidney disease. Pregnancy, puberty, and cataract surgery can accelerate these changes.

A nurse on a medical unit is providing care for a 37-year-old female client who has a diagnosis of Graves disease. Which assessments should the nurse prioritize?

Eye health and visual acuity Explanation: The ophthalmopathy of Graves disease can cause severe eye problems, including tethering of the extraocular muscles resulting in diplopia; involvement of the optic nerve, with some visual loss; and corneal ulceration because the lids do not close over the protruding eyeball (due to the exophthalmos). Eye assessment is consequently a priority over assessment of skin integrity, cognition, or musculoskeletal status.

The physiologic rationale for hanging normal saline (0.9% NS) or 5% dextrose in water (D5W) to a client who has been experiencing diarrhea includes:

Facilitating the absorption of osmotically active particles Explanation: Water absorption from the intestine is linked to absorption of osmotically active particles such as glucose and sodium. It follows that an important consideration in facilitating the transport of water across the intestine (and decreasing diarrhea) after temporary disruption in bowel function is to include sodium and glucose in the fluids that are consumed. None of the other distractors addresses this principle. Activating the pancreatic enzymes of trypsin and elastase is needed for protein digestion and absorption. Emulsification of fats begins in the stomach and continues in the duodenum under the influence of bile from the liver.

The nurse is teaching her client with hepatobiliary disease about her diet. She tells her that she may have steatorrhea, which is the malabsorption of which dietary component?

Fat Explanation: The conditions that impair one or more steps involved in digestion and absorption of nutrients can be divided into three broad categories: intraluminal maldigestion, disorders of transepithelial transport, and lymphatic obstruction. Hepatobiliary disease is a common cause of intraluminal maldigestion. Fats are not absorbed in the upper jejunum and the excretion of fat in the stool is steatorrhea.

Diabetic ketoacidosis (DKA) in a client with type 1 diabetes occurs when the lack of insulin leads to the release of which physiologic product?

Fatty acids Explanation: DKA most commonly occurs in type 1 diabetes, when the lack of insulin leads to unsuppressed adipose cell lipase activity that breaks down triglycerides into fatty acids and glycerol. The subsequent increase in fatty acid levels leads to ketone production by the liver. Serum potassium levels may be normal or elevated, despite total potassium depletion resulting from protracted polyuria and vomiting. Metabolic acidosis is caused by the excess ketoacids that require buffering by bicarbonate ions; this leads to a marked decrease in serum bicarbonate levels. Stress increases the release of cortisol and other gluconeogenic hormones and predisposes the person to the development of ketoacidosis.

A teenager has been diagnosed with failure to thrive possibly due to malabsorption syndrome. In addition to having diarrhea and bloating, the client more than likely has what hallmark manifestation of malabsorption?

Fatty, yellow-gray, foul-smelling stools Explanation: General symptoms of malabsorption syndrome include diarrhea, flatulence, bloating, cramping, and weight loss. A hallmark of malabsorption is steatorrhea, characterized by fatty, yellow-gray, and foul-smelling stools. Feeling there is incomplete emptying of the bowel is one of the signs and symptoms of colon cancer. Abdominal distention occurs with many gastrointestinal diseases and is not specific to malabsorption syndrome. Esophageal reflux with heartburn is usually associated with gastroesophageal reflux disease.

A teenager who has a history of achalasia will likely complain of which clinical manifestation?

Feeling like there is food stuck in the back of the throat Explanation: Achalasia produces functional obstruction of the esophagus so that food has difficulty passing into the stomach, and the esophagus above the lower esophageal sphincter becomes distended. Symptoms following high-fat intake are usually associated with gallbladder disease. Projectile vomiting is usually related to increased intracranial pressure. Vomiting blood can be associated with esophagitis, erosion of the esophagus, bleeding esophageal varices, or esophageal cancer.

Which hormone is secreted based on a cyclic rather than a diurnal manner?

Follicle-stimulating hormone (FSH) Explanation: Hormone secretion varies widely over a 24-hour period. Some hormones, such as GH and adrenocorticotropic hormone (ACTH), have diurnal fluctuations that vary with the sleep-wake cycle. Others, such as the female sex hormones (e.g., follicle-stimulating hormone [FSH]) are secreted in a complicated cyclic manner. The levels of hormones such as insulin and antidiuretic hormone (ADH) are regulated by feedback mechanisms that monitor substances such as glucose (insulin) and water (ADH) in the body. The levels of many of the hormones are regulated by feedback mechanisms that involve the hypothalamic-pituitary-target cell system.

A pharmacology student is studying gastrointestinal medications. The student explains that proton pump inhibitors (PPIs) are used to inhibit secretion of:

Gastric acid Explanation: PPIs (also known as the H+/K+-ATPase transporter), which are used in the treatment of acid reflux and peptic ulcer, inhibit gastric acid secretion. The other options are not inhibited by PPIs.

A student is comparing the actions of the gastrointestinal hormones. Which hormone acts as an appetite-stimulating signal?

Ghrelin Explanation: Ghrelin stimulates secretion of growth hormone and acts as a appetite-stimulating signal from the stomach. GLP-1 decreases appetite. The other options do not have an effect on appetite.

The nurse is caring for a client who received regular insulin at 7 am. Four hours later the nurse finds the client diaphoretic, cool, and clammy. Which of these interventions is the priority?

Give the client a concentrated carbohydrate. Explanation: The client is displaying symptoms of hypoglycemia, which include headache, difficulty in problem solving, altered behavior, coma, and seizures. Hunger may occur. Activation of the sympathetic nervous system may cause anxiety, tachycardia, sweating, and cool and clammy skin.

The family of a client in the hospital with diabetes mellitus that is out of control asks the nurse to explain the client's recent weight loss while eating more than usual. How will the nurse respond?

Glucose is unused without insulin, so body fats are used for energy. Explanation: Most of these options are true statements, but they do not answer the question asked by the family. Normally, nutrients are metabolized in a number of ways. Glucose is transported into cells by insulin and then is broken down to carbon dioxide and water. When there is surplus glucose present, it is metabolized and stored as glycogen in the liver and skeletal muscles. Further surplus is converted by the liver to fatty acids and stored as triglycerides. When triglycerides are metabolized, the glycerol molecule enters the glycolytic pathway to release energy. Excess proteins are also converted to fatty acids for storage. Insulin is needed to transport glucose into cells, prevent fat breakdown, and inhibit gluconeogenesis. When diabetes is out of control there is lack of insulin. Weight loss occurs as the cells break down fats to use fatty acids for energy.

A nurse assessing an 8-year-old child notes that the child is 6 feet tall. Which diagnostic test will be performed to evaluate the cause of the condition?

Glucose load Explanation: Endocrine testing may include urine collection, blood levels, or tests of stimulation or suppression. A child exhibiting signs of gigantism requires the glucose load test. If the pituitary responds correctly, the GH level will drop. If there is a secreting tumor, the GH level will increase.

Which hormone is produced by the anterior pituitary gland?

Growth hormone (GH) Explanation: GH is among the hormones produced and released by the anterior pituitary. Oxytocin is a posterior pituitary hormone, whereas CRH is produced by the hypothalamus. Norepinephrine and epinephrine are produced by the adrenal medulla.

In the balance of secretions in the gastric mucosa by the parietal cells, which ion is produced to buffer the production of hydrochloric acid?

HCO3- Explanation: Normally the secretion of hydrochloric acid by the parietal cells of the stomach is accompanied by secretion of bicarbonate ions (HCO3-), which protects the mucosa from injury, as long as they are produced in equal amounts.

A 62-year-old man who is overweight has just been diagnosed with type 2 diabetes. The nurse educator is instructing him in the ways his diabetes can be controlled. The nurse should initially prioritize which action?

Helping the client make meaningful changes to his diet and activity level. Explanation: Weight loss and dietary management are the initial focus of treatment for type 2 diabetes. For many people with type 2 diabetes, the benefits of exercise include a decrease in body fat, better weight control, and improvement in insulin sensitivity. If good glycemic control cannot be achieved with exercise and diet, then antidiabetic agents and even insulin can be added to the treatment plan. Education is imperative, but there is no need to emphasize hypoglycemia, since there is no evidence the client is on a medication that would cause hypoglycemia.

A neonate is undergoing phototherapy for the treatment of jaundice and accompanying high levels of bilirubin. Which causes are the likely factors responsible for neonatal hyperbilirubinemia? Select all that apply.

High bilirubin production Limited ability to excrete bilirubin Explanation: In infants, hyperbilirubinemia results from high bilirubin production coupled with a limited ability to excrete it. Hemolysis is not usually the cause of the problem in infants. Maternal bilirubin is not passed on to infants and neonatal cholestasis is not a recognized phenomenon.

A client is diagnosed with Crohn disease. The nurse instructs the client on which type of dietary needs?

High-calorie, vitamin, and protein diet Explanation: Nutritional deficiencies are common in Crohn disease because of diarrhea, steatorrhea, and other malabsorption problems. A nutritious diet that is high in calories, vitamins, and proteins is recommended. Because fats often aggravate the diarrhea, it is recommended they be avoided. Elemental diets, which are nutritionally balanced but residue-free and bulk-free, may be given during the acute phase of the illness.

Which substance necessary for vitamin B12 absorption is produced by the parietal cells in the stomach?

Intrinsic factor Explanation: Intrinsic factor is produced by the parietal cells in the stomach and is necessary for the absorption of vitamin B12. Hydrochloric acid activates pepsinogen into the enzyme pepsin, which then helps digestion by breaking the bonds linking amino acids, a process known as proteolysis. An enzyme present in red blood cells, carbonic anhydrase, aids in the conversion of carbon dioxide to carbonic acid and bicarbonate ions.

Which physiologic process allows hormones to exert influence upon some cells and not others?

Hormone receptors Explanation: Hormone receptors are complex molecular structures (usually proteins) that are located either on the cell surface or inside target cells. The structure of these receptors is specific to a particular hormone, which allows target cells to respond to one hormone and not to others. For example, receptors in the thyroid are specific for thyroid-stimulating hormone, and receptors on the gonads respond to the gonadotropic hormones. Positive feedback control occurs when rising levels of a hormone cause another gland to release a hormone that is stimulating to the first. Protein binding describes the ability of proteins to form bonds with other substances. Pituitary-hypothalamic feedback describes negative feedback.

When hypofunction of an endocrine organ is suspected, which type of diagnostic test can be administered to measure and assess target gland response?

Hormone stimulation Explanation: Stimulating hormone can be given to identify (determine hypofunction) if the target gland is able to increase hormone response to increased stimulation. Agglutination with enzymes is a way of measuring hormone antigen levels. The 24-hour urine sample measures hormone metabolite excretion. Radioactive hormone-antibody binding levels provide a method of measuring plasma levels.

The nurse is caring for a client with right upper quadrant pain secondary to acute choledocholithiasis. If the common bile duct becomes obstructed, which manifestation will the nurse expect?

Hyperbilirubinemia Explanation: Choledocholithiasis, stones in the common duct, usually originate in the gallbladder but can form spontaneously in the common duct. Bilirubinuria and an elevated serum bilirubin (hyperbilirubinemia) are present if the common duct is obstructed. With acute cholecystitis, approximately 75% of clients have vomiting. Ascites is common with late-stage liver failure rather than duct obstructions. Bleeding is associated with liver failure due to deficiency of clotting factors, and is also associated with acute pancreatitis due to activated enzymes causing fat necrosis and hemorrhage from the necrotic vessels.

When caring for the client with acute pancreatitis, which alterations does the nurse recognize is consistent with the disease?

Hyperglycemia Explanation: Serum amylase and lipase are the laboratory markers most commonly used to establish a diagnosis of acute pancreatitis. The white blood cell count may be increased, and hyperglycemia and an elevated serum bilirubin level may be present.

The nurse is caring for the client with pancreatic cancer. The nurse monitors the client for which complication?

Hyperglycemia due to inability to synthesize insulin Explanation: The endocrine pancreas supplies the insulin needed to lower glucose levels in the blood; damage to the pancreas may alter this function, causing hyperglycemia.

A client asks the nurse what part of the brain regulates appetite. Which is the best response by the nurse?

Hypothalamus Explanation: Appetite or the desire for food is regulated by the hypothalamus and other associated parts of the brain. The medulla deals with the autonomic (involuntary) functions of breathing, heart rate, and blood pressure. The midbrain acts as a relay information system for the auditory, visual, and motor systems of the body. The cerebellum coordinates voluntary movements, posture, and balance in humans.

Which gland acts as a signal relaying bridge between multiple body systems and the pituitary gland?

Hypothalamus Explanation: The activity of the hypothalamus is regulated by both hormonally mediated signals (e.g., negative feedback signals) and by neuronal input from a number of sources. Neuronal signals are mediated by neurotransmitters such as acetylcholine, dopamine, norepinephrine, serotonin, gamma-aminobutyric acid (GABA), and opioids. Cytokines that are involved in immune and inflammatory responses, such as the interleukins, also are involved in the regulation of hypothalamic function. This is particularly true of the hormones involved in the hypothalamic-pituitary-adrenal axis. Thus, the hypothalamus can be viewed as a bridge by which signals from multiple systems are relayed to the pituitary gland. This cannot be said of the other options.

Release and synthesis of anterior pituitary hormones are mainly regulated by which part of the body?

Hypothalamus Explanation: The hypophysis (pituitary plus hypothalamus) and hypothalamus stimulatory hormones regulate the release and synthesis of anterior pituitary hormones. The adrenal gland and thymus gland hormones are regulated by the hypothalamus. Cell receptors are involved with the target cell response to the hormones.

An infant whose mother had myxedema during the pregnancy has failed to meet standards for growth and is developmentally delayed. Which hormonal imbalance is this child exhibiting?

Hypothyroidism Explanation: Thyroid hormone is necessary for metabolism at all ages, as well as growth and development during childhood. Uncorrected thyroid insufficiency in childhood leads to cretinism, a condition with marked physical and intellectual disability. Myxedema is the term used for thyroid insufficiency in adults.

The nurse is caring for the client with hepatocellular carcinoma. What does the nurse recognize is a cause of this disease?

Illness with hepatitis B or C Explanation: With hepatitis B virus and hepatitis C virus, both of which become integrated into the host DNA, repeated cycles of cell death and regeneration afford the potential for development of cancer-producing mutations.

When caring for the client with Laennec cirrhosis, the nurse recognizes which pathophysiologic finding to be an expected etiology of jaundice?

Impaired uptake of bilirubin Explanation: The five major causes of jaundice are excessive destruction of red blood cells, impaired uptake of bilirubin by the liver cells, decreased conjugation of bilirubin, and obstruction of bile flow in the canaliculi of the hepatic lobules or in the intrahepatic or extrahepatic bile ducts, and excessive extrahepatic production of bilirubin. Hypoxia is not a factor in jaundice.

The health care provider has completed the assessment on a client with jaundice. The provider determines that which pathophysiologic abnormality could cause the jaundice?

Impaired uptake of bilirubin by the liver Explanation: The five major causes of jaundice are (1) excessive destruction of RBCs, (2) impaired uptake of bilirubin by the liver cells, (3) decreased conjugation of bilirubin, (4) obstruction of the bowel flow in the canaliculi of the hepatic lobules or in the intrahepatic or extrahepatic bile ducts, and (5) excessive extrahepatic production of bilirubin. Jaundice would not occur as a result of decreased oxygen. Jaundice in newborns is the result of the breakdown of fetal hemoglobin.

During assessment of a recently admitted client with right-sided heart failure, the nurse notes that the liver is enlarged and verifies by palpation and percussion. A student asks the nurse to explain how the liver and heart failure are related. Which is the nurse's best response?

In right-sided heart failure, there is high pressure in the inferior vena cava that backlogs blood into the liver. Explanation: In right-sided heart failure in which the pressure in the vena cava increases, blood backs up and accumulates in the liver. The pressure difference between the hepatic vein and the portal vein normally is such that the liver stores approximately 450 mL of blood. Right-sided heart failure backs up the venous side of circulation whereas left-sided failure backlogs blood into the lungs. The lobules are the functional units of the liver. The classic lobule consists of stacks of anastomosing plates of hepatocytes one cell thick. Each lobule is organized around a central vein that empties into the hepatic veins and from there into the inferior vena cava.

When the nurse is performing a health history for a client who is being admitted for hyperthyroidism, what symptom does the client report that the nurse would find associated with this disorder?

Increase in appetite Explanation: Thyroid hormone enhances gastrointestinal function, causing an increase in motility and production of GI secretions that often results in diarrhea. An increase in appetite and food intake accompanies the higher metabolic rate that occurs with increased thyroid hormone levels. At the same time, weight loss occurs because of the increased use of calories.

A client is admitted to the medical surgical unit with a history of inflammatory bowel disease. The nurse knows that the clinical manifestations of both Crohn disease and ulcerative colitis are the result of activation of which cells?

Inflammatory Explanation: The term "inflammatory bowel disease" is used to designate two related inflammatory intestinal disorders: Crohn disease and ulcerative colitis. Both diseases produce inflammation of the bowel. The clinical manifestations of both Crohn disease and ulcerative colitis are ultimately the result of inflammatory cells with elaboration of inflammatory mediators that cause nonspecific tissue damage.

Crohn disease is treated by several measures. Treatment with sulfasalazine will focus on which aspect of this disease?

Inflammatory suppression Explanation: Treatment methods focus on terminating the inflammatory response and promoting healing, maintaining adequate nutrition, and preventing and treating complications. Several medications have been successful in suppressing the inflammatory reaction, including corticosteroids, sulfasalazine, metronidazole, azathioprine, 6-mercaptopurine, methotrexate, and infliximab.

Which metabolic abnormality can lead to the development of type 2 diabetes?

Insulin resistance Explanation: The metabolic abnormalities that lead to type 2 diabetes include (1) peripheral insulin resistance, (2) deranged secretion of insulin by the pancreatic beta cells, and (3) increased glucose production by the liver. Obese people have increased resistance to the action of insulin and impaired suppression of glucose production by the liver, resulting in both hyperglycemia and hyperinsulinemia. Lifestyle and overeating seem to be the triggering events (rather than metabolic abnormalities). Acute pancreatitis is a reversible impairment of alpha and beta cell function, with hypoinsulinemia as a complication.

The liver has many jobs. One of the most important functions of the liver is to cleanse the portal blood of old and defective blood cells, bacteria in the bloodstream, and any foreign material. Which cells in the liver are capable of removing bacteria and foreign material from the portal blood?

Kupffer cells Explanation: Kupffer cells are reticuloendothelial cells that are capable of removing and phagocytizing old and defective blood cells, bacteria, and other foreign material from the portal blood as it flows through the sinusoid. Langerhans cells are stellate dendritic cells found mostly in the stratum spinosum of the epidermis. Epstein cells do not exist. Davidoff cells are large granular epithelial cells found in intestinal glands.

When caring for the client with portal hypertension and ascites, which dietary intervention does the nurse suggest to prevent the progression of fluid accumulation?

Limit intake of sodium. Explanation: Treatment of ascites usually focuses on dietary restriction of sodium, administration of diuretics, and possible fluid restriction.

Which combination of insulins matches most closely the normal pattern of release by the pancreas?

Lispro insulin with each meal plus glargine at bedtime Explanation: The pancreas responds to glucose in two ways: a short rapid response followed by a longer, steady response. The insulin pattern that most closely matches this is to administer a rapid-acting insulin, such as lispro, with the long-acting insulin glargine. The other two categories of insulin are short and intermediate. They have a slower onset, peak, and duration that do not match the natural release of insulin in response to food. Clients need to coordinate meals to match the insulin activity.

While teaching a science class, the instructor mentions that both autocrine and paracrine hormonal actions occur without entering the bloodstream. A student asks, "What cells do paracrine actions affect?" Which response is correct?

Local Explanation: Paracrine actions are hormonal interactions with local cells other than those that produce the hormone; autocrine actions are with self-cells (cells from which they were produced). Both autocrine and paracrine hormonal actions affect target cells. Neither paracrine nor autocrine actions affect cell storage.

Which factor contributes to an infant developing macrosomia (large body size)?

Maternal diabetes Explanation: A mother with diabetes during pregnancy is more likely to have a larger fetus, especially if the diabetes is uncontrolled. The other factors may cause the infant to be small for gestational age (SGA).

Which maternal factor contributes to an infant having macrosomia, hypoglycemia, and hyperbilirubinemia?

Maternal diabetes Explanation: A mother with diabetes is more likely to have a large body size (macrosomia), especially if the diabetes is uncontrolled. Other fetal abnormalities include hypoglycemia, hypocalcemia, polycythemia, and hyperbilirubinemia. The other factors contribute primarily to the infant being small for gestational age (SGA). The other disorders do not necessarily lead to a high birth weight.

The nurse is caring for a client with asterixis. Which assessment should the nurse make to help a diagnosis of hepatic encephalopathy?

Mental status Explanation: Hepatic encephalopathy refers to the totality of central nervous system manifestations of liver failure. It is characterized by neural disturbances ranging from a lack of mental alertness to confusion, coma, and convulsions. A very early sign of hepatic encephalopathy is a flapping tremor called asterixis.

Which treatment regimen is most likely to result in stable blood glucose levels for a client with a diagnosis of type 1 diabetes?

Monitor blood glucose levels throughout the day and administer exogenous insulin replacement as needed Explanation: Because of the loss of insulin response, all people with immune-mediated type 1 diabetes require exogenous insulin replacement to reverse the catabolic state, control blood glucose levels, and prevent ketosis.

A client with a history of brain tumors that resulted in partial removal of the pituitary gland years ago expresses concern to the health care provider about whether she will be able to breast-feed her infant. Which physiologic function of the pituitary gland facilitates breast milk production?

Prolactin Explanation: The anterior pituitary gland or adenohypophysis contains five cell types: (1) thyrotrophs, which produce thyrotropin, also called TSH; (2) corticotrophs, which produce corticotropin, also called ACTH; (3) gonadotrophs, which produce the gonadotropins, LH and FSH; (4) somatotrophs, which produce GH; and (5) lactotrophs, which produce prolactin that is involved with breast growth and milk production.

An ultrasound confirms appendicitis as the cause of a client's sudden abdominal pain. Which etiologic process is implicated in the development of appendicitis?

Obstruction of the intestinal lumen Explanation: Appendicitis is thought to be related to intraluminal obstruction with a fecalith, gallstones, tumors, parasites, or lymphatic tissue. Osmotic and bacterial changes are not thought to induce appendicitis, and the intestinal mucosa does not slough off either before or during episodes of appendicitis.

When assessing a client with acute cholecystitis, the nurse anticipates the client's report of pain will be consistent with which description?

Pain in the right upper quadrant referred to the same shoulder Explanation: The pain of biliary colic begins abruptly and increases in intensity. It is usually located in the upper right quadrant or epigastric area and may be referred to the upper back, the right shoulder, or midscapular region.

Hormones that cause the subsequent stimulation or release of another hormone, such as thyroid-stimulating hormone, typically belong to which structural classification?

Peptide proteins and glycoproteins Explanation: Hormones whose effect includes the stimulation and release of other hormones belong to the structural category of peptide proteins and glycoproteins.

A client has been admitted to the hospital with an exacerbation of peptic ulcer disease. The nurse is aware the client is at risk for:

Perforation Hemorrhage Obstruction Explanation: The most common complications of peptic ulcer are hemorrhage, perforation, and gastric outlet obstruction. Weight gain and increased urinary output would not occur as the client may experience volume loss.

Which layer of the digestive tract constitutes the outer wall of the intestine and contains a serous fluid between its two layers?

Peritoneum Explanation: The peritoneum is the largest serous membrane and constitutes the outer wall of the intestine, continuous with the mesentery. The greater omentum helps to prevent infection from entering the peritoneal cavity, and protects the intestines from cold. Haustration is the segmental mixing of movements of the large intestine (colon).

A client is admitted with an abrupt onset of referred pain to the epigastric area, with an episode of nausea. On the nurse's initial assessment, the client is lying still and taking shallow breaths, with a rigid abdomen. Which problem is the client experiencing?

Peritonitis Explanation: The onset of peritonitis may be acute, as with a ruptured appendix, or it may have a more gradual onset, as occurs in pelvic inflammatory disease. The pain usually is more intense over the inflamed area. The person with peritonitis usually lies still because any movement aggravates the pain. Breathing often is shallow to prevent movement of the abdominal muscles. The abdomen usually is rigid and sometimes described as boardlike because of reflex muscle guarding.

When teaching a group of nursing students about the liver, the nurse relates that Kupffer cells function to remove harmful substances or cells from the portal blood and venous sinusoids through which process?

Phagocytosis Explanation: Kupffer cells, which line the venous sinusoids, are reticuloendothelial cells that are capable of removing, engulfing, and phagocytizing old and defective blood cells, bacteria, and other foreign material from the portal blood as it flows through the sinusoid. This phagocytic action removes the enteric bacilli and other harmful substances that have filtered into the blood from the intestine. Kupffer cells do not have cytotoxic, osmotic, or ultrafiltration capabilities.

Which gland is often referred to as the master gland because it secretes many hormones?

Pituitary Explanation: The pituitary gland has been called the master gland because its hormones control the functions of many target glands and cells. That is not a term used to refer to the other options.

Which structure controls the functions of the greatest number of target glands and cells?

Pituitary gland Explanation: The pituitary gland has been called the master gland because its hormones control the functions of many target glands and cells. It supersedes the importance of the thyroid, adrenal cortex, or pancreas in this regulatory role.

Select the most common symptoms of diabetes. Select all that apply.

Polydipsia Polyuria Polyphagia Explanation: The most commonly identified signs and symptoms of diabetes are often referred to as the three polys: (1) polyuria (i.e., excessive urination), (2) polydipsia (i.e., excessive thirst), and (3) polyphagia (i.e., excessive hunger). Polyhydramnios is a medical condition describing an excess of amniotic fluid in the amniotic sac. Polycythemia is a condition of increased red blood cells.

A client receives steroids for several months to treat an inflammatory condition. Which action by the primary health care provider indicates an understanding of the negative feedback mechanism when the client no longer needs the medication?

Prescribing a tapering dose of the medication over weeks Explanation: Negative feedback occurs when secretion of one hormone causes a reduction in the secretion of the hormone that stimulates production of the first hormone. In this case, adrenocorticotropic hormone (ACTH) manufactured by the anterior pituitary would normally stimulate release of cortisol, but with the increase of cortisol produced by the secreting tumor, enough cortisol already floods the system that there should be a reduction in the ACTH level. Sudden withdrawal of the medication would leave the client without glucocorticoids and risk an Addisonian crisis. Gradual withdrawal of the medication allows the pituitary to measure the drop in cortisol levels and begin secreting ACTH.

A hospital client with a diagnosis of type 1 diabetes has been administered a scheduled dose of regular insulin. Which effect will result from the action of insulin?

Promotion of glucose uptake by target cells Explanation: The actions of insulin are threefold: (1) it promotes glucose uptake by target cells and provides for glucose storage as glycogen; (2) it prevents fat and glycogen breakdown; and (3) it inhibits gluconeogenesis and increases protein synthesis. Glucagon, not insulin, promotes glycogenolysis.

What is the primary purpose of the mucosal barrier in the gastrointestinal tract?

Protection from acid secretion Explanation: The primary purpose of gastric mucosa is resistance to the highly acidic secretion that it produces.

Which statement is true concerning food digestion?

Protein digestion begins in the stomach. Explanation: Protein digestion begins in the stomach with the action of pepsin. Pepsin cannot act in the small intestine, as it is broken down by the alkaline pH in the gastro intestinal tract. Proteins do not break down carbohydrates, and monosaccharides can be absorbed.

An adult client has been diagnosed with gastroesophageal reflux disease (GERD) after reporting ongoing "heartburn" unrelieved with antacids. Which medication should the nurse anticipate being prescribed for this client?

Proton pump inhibitor (PPI) Explanation: PPIs act by inhibiting the gastric proton pump, which regulates the final pathway for acid secretion. Lactulose is prescribed to treat or prevent complications of liver disease (hepatic encephalopathy). It does not cure the problem, but may help to improve mental status. Lactulose is a colonic acidifier that works by decreasing the amount of ammonia in the blood. Antibiotics are used to treat Helicobacter pylori infections commonly found in peptic ulcer disease. NSAID use should be avoided when possible.

An anatomy student explains that the funnel-shaped portion of the stomach that connects with the intestine is called which region?

Pyloric region Explanation: The funnel-shaped portion that connects with the small intestine is called the pyloric region. The small part of the stomach that surrounds the cardiac orifice is called the cardiac region. The dome-shaped region that bulges above the cardiac region is called the fundus, and the middle portion is called the body.

The circular layer of smooth muscle that lies between the stomach and the small intestine is called:

Pyloric sphincter Explanation: At the end of the pyloric channel, the circular layer smooth muscle thickens to form the pyloric sphincter. This muscle serves as a valve that controls the rate of stomach emptying and prevents the regurgitation of intestinal contents back into the stomach. There is no cardiac sphincter in the GI tract. The antrum is a portion of the stomach that is the wider, upper portion of the pyloric region. The cardiac orifice is the opening between the esophagus and the stomach.

Regurgitation of bile salts and duodenal contents can lead to gastric ulcers. Which structure prevents this from happening?

Pyloric sphincter Explanation: The pyloric sphincter prevents the backflow of gastric contents and allows them to flow into the duodenum at a rate commensurate with the ability of the duodenum to accept them. This is important because the regurgitation of bile salts and duodenal contents can lead to gastric ulcers. The esophagus opens into the stomach through an opening called the cardiac orifice. The upper esophageal sphincter is the pharyngoesophageal sphincter; the lower esophageal sphincter is the gastroesophageal sphincter.

A woman in her 28th week of pregnancy tests positive for gestational diabetes mellitus and begins to follow a nutritional plan at home. What result at the follow-up visit indicates a successful outcome?

Random blood glucose 85 mg/dL (4.72 mmol/L) Explanation: The goals of the nutritional plan for gestational diabetes mellitus (GDM) include normal glucose levels, no ketosis, proper weight gain for the pregnancy, and adequate nutrition for fetal health.

A nurse is completing an abdominal assessment on a client suspected to have appendicitis. When the nurse applies and then releases pressure in the client's right lower quadrant, the client experiences tenderness. The nurse is documenting the presence of:

Rebound tenderness Explanation: The nurse documents the presence of rebound tenderness, defined as tenderness that occurs when the nurse applies and then releases pressure to an area.

A client presents to the emergency room with fatigue, weakness, dehydration, and thirst. What additional symptoms would correlate with a diagnosis of type 1 diabetes mellitus? Select all that apply.

Recent weight loss Polyuria Blurred vision Explanation: The cardinal manifestations of diabetes mellitus are the three "polys": polyuria, polydipsia, and polyphagia, and weight loss. Additional symptoms include fatigue and weakness, blurred vision, and skin infections.

The results of a client's 24-hour stool specimen indicate 20 g or more of fat. The nurse would interpret this as:

Steatorrhea Explanation: Steatorrhea is the term used to describe fatty stools. It usually indicates that there is 20 g or more of fat in a 24-hour stool sample

Hormones are chemical messengers that provide which function in the body?

Regulate body functions Explanation: Hormones regulate and integrate body functions. Hormones act on specific target cells, but they cause a variety of effects on tissues. Hormones do not transport other substances; hormones are transported and present in body fluids at all times.

The nurse is performing a physical assessment of the gastrointestinal tract. In which area does the nurse place the hands for palpation of the liver?

Right upper quadrant Explanation: The liver is the largest visceral organ in the body, located below the diaphragm in the right hypochondrium or right upper quadrant.

A client with severe hypothyroidism is presently experiencing hypothermia. What nursing intervention is a priority in the care of this client?

Slow rewarming of the client to prevent vasodilation and vascular collapse Explanation: If hypothermia is present, active rewarming of the body is contraindicated because it may induce vasodilation and vascular collapse. Prevention is preferable to treatment and entails special attention to high risk populations, such as women with a history of Hashimoto thyroiditis.

A client is admitted to the hospital for further evaluation of problems experienced with gastrointestinal digestion and absorption. The nurse anticipates the health care provider will further assess the:

Small intestine Explanation: Digestion and absorption of nutrients take place primarily in the small intestine.

Villi are anatomic features that contribute to the enlarged surface area of the:

Small intestine Explanation: The distinguishing characteristic of the small intestine is its large surface area. Anatomic features that contribute to this enlarged surface area are the circular folds that extend into the lumen of the intestine and the villi.

A nurse explains to her client that food is moved along the gastrointestinal (GI) tract with intermittent contractions that mix the food and move it along. These movements are found in which organ?

Small intestine Explanation: The movements of the GI tract can be either rhythmic or tonic. Rhythmic movements consist of intermittent contractions that are responsible for mixing and moving food along the digestive tract. They are found in the esophagus, the antrum of the stomach, and the small intestine. Tonic movements are found in the lower esophagus, the upper part of the stomach, the ileocecal valve, and the internal anal sphincter.

A client with a peptic ulcer should be assessed by the nurse for which possible cause? Select all that apply.

Smoking history Family history Alcohol usage Explanation: Epidemiologic studies have identified independent factors that augment the effect of H. pylori infection and NSAID-produced peptic ulcer disease. The factors include advancing age, a prior history of peptic ulcer, multiple NSAID use, and concurrent use of warfarin (an anticoagulant) and corticosteroid drugs. Smoking may augment the risk of peptic ulcer by impairing healing. Alcohol use may cause increased acid production. There is no convincing evidence that dietary factors play a role in development of peptic ulcer. There is increased incidence of peptic ulcer in families.

The nurse teaches the client that which of these contributed to the development of acute cholelithiasis?

Stasis of bile Explanation: Two primary factors contribute to the formation of gallstones: abnormalities in the composition of bile and the stasis of bile (rather than rapid elimination). Inflammation of the gallbladder alters the absorptive characteristics of the mucosal layer, allowing excessive absorption of water and bile salts. Although a number of factors are associated with the development of acute pancreatitis, most cases result from gallstones (rather than cause gallstone formation) or alcohol use disorder. Alcohol is known to be a potent stimulator of pancreatic secretions, and it also is known to cause partial obstruction of the sphincter of the pancreatic duct; alcohol intake is not a factor in the development of cholesterol or bilirubin stones.

Which clinical manifestation would lead the nurse to suspect the client has malabsorption syndrome with a deficiency in fat absorption?

Steatorrhea Explanation: In malabsorption syndrome, there is loss of fat in the stools and failure to absorb the fat-soluble vitamins. This can result in weight loss, steatorrhea, and fat-soluble vitamin deficiency. Cramping is associated with water/electrolyte imbalances. Eye problems like dry eyes is due to malabsorption of vitamin A. Glossitis is associated with folic acid deficiency.

The kidney produces 1,25-dihydroxyvitamin D. This form of vitamin D is responsible for which action in the body?

Stimulates calcium absorption from the intestine Explanation: 1,25-dihydroxyvitamin D from the kidney stimulates calcium absorption from the intestine. Somatostatin from the pancreas delays intestinal absorption of glucose. Thyroxine (T4) can increase metabolic rate. Growth hormone (GH) stimulates growth of bone and muscle.

Gastrin production, a task that is performed by the stomach, results in which effect?

Stimulation of gastric acid secretion by parietal cells Explanation: The primary function of gastrin is the stimulation of gastric acid secretion. It does not have a direct effect on exocrine or endocrine pancreatic function, and it does not participate actively in the digestion of starches.

A client has received an injection containing thyrotropin-releasing hormone (TRH) and is now being assessed for serum levels of thyroid-stimulating hormone (TSH). Which type of diagnostic testing is this client undergoing?

Stimulation testing Explanation: Introduction of TRH tests the pituitary gland's ability to produce TSH, and is an example of a stimulation test. Suppression testing examines a gland's response to a stimulus that would normally result in decreased hormone production. RIA and autoantibody testing are examples of direct and indirect measurement of serum levels of a hormone.

The physician suspects a client may be experiencing hypofunction of an endocrine organ. Select the most appropriate test to determine organ function.

Stimulation tests Explanation: Stimulation tests are used when hypofunction of an endocrine organ is suspected. Suppression tests are used when hyperfunction of an endocrine organ is suspected. Genetic testing is used for DNA analysis, and imaging may be used as a follow-up after the diagnosis.

The production of chyme occurs in which organ?

Stomach Explanation: The chemical breakdown of protein begins in the stomach where food is converted to a creamy mixture called chyme.

A client has been diagnosed with an incompetent pyloric sphincter. The nurse educating the client mentions that this is due to abnormal communication between which components of the gastrointestinal tract?

Stomach and duodenum Explanation: The pylorus is located between the body of the stomach and the duodenum.

The cause of gastric carcinomas has been influenced by which factors? Select all that apply.

Strain of Helicobacter pylori (H. pylori) Environmental factors Family history Explanation: Chronic infection with H. pylori appears to serve as a cofactor in some types of gastric carcinomas. In addition to genetics, the likelihood of developing gastric cancer from an H. pylori infection is related to the strain of H. pylori infection, environmental factors, and duration of infection.

The client who has experienced third-degree burns is susceptible to which specific type of gastrointestinal (GI) ulceration?

Stress Explanation: Stress ulcers refer to GI ulcerations that develop in people with large-surface-area burns.

A nurse administering a client's medication tells the client that a proton pump inhibitor has been added. When the client asks the purpose of the medication, the nurse responds that it is to prevent:

Stress ulcer Explanation: Proton pump inhibitors are the first line of medications used in the prevention of stress ulcers.

Following the analysis of colonoscopy with biopsy, a client is diagnosed with colorectal cancer. Which treatment modality will be the mainstay of this client's treatment regimen?

Surgery Explanation: The only recognized treatment for cancer of the colon and rectum is surgical removal. Postoperative radiation therapy may be used and has in some cases demonstrated increased 5-year survival rates. Postoperative adjuvant chemotherapy may be used. Radiation therapy and chemotherapy are used as palliative treatment methods as well.

Which type of nervous system response can cause a marked decrease in mucous production, leaving the area susceptible to irritation, thereby causing ulcer development in the duodenum?

Sympathetic stimulation Explanation: Brunner glands secrete large amounts of alkaline mucus that protects the duodenum from digestive enzymes. They are influenced by autonomic nervous system activity. For example, sympathetic stimulation causes a marked decrease in mucous production, leaving this area more susceptible to irritation. As a result, ulcers are four times more likely to occur in the duodenum than in the stomach.

A client has developed the facial appearance that is characteristic of myxedema, along with an enlarged tongue, bradycardia, and voice changes. Which treatment modality is most likely to benefit this client?

Synthetic preparations of T3 or T4 Explanation: Myxedema and the client's other signs are associated with hypothyroidism, which necessitates thyroid hormone replacement. Beta-adrenergic blocking drugs and antithyroid drugs are indicated in the treatment of hyperthyroidism, whereas treatments relevant to adrenal cortical function are not relevant to hypothyroidism.

A client is diagnosed with choledocholithiasis and acute suppurative cholangitis and is being rushed to surgery to prevent which possible complication?

The accumulation of purulent bile, which can cause sepsis. Explanation: Complications of choledocholithiasis include acute suppurative cholangitis when purulent bile fills and distends bile ducts. It is characterized by the presence of an altered sensorium, lethargy, and septic shock. Bilirubinuria and malabsorption are not emergency situations. This situation is not part of the etiology of hepatitis.

A client is managing diabetes with exercise and diet. The health care provider reviews the client's most recent laboratory results: fasting blood glucose level at 80 mg/dL (4.44 mmol/L) and a hemoglobin A1C of 5% (0.05). Select the response that best identifies the client.

The client is achieving normal glycemic control. Explanation: The reading for the fasting blood glucose is appropriate, and the hemoglobin A1C level estimates good control of glucose levels over a 6- to 12-week period. This client should continue the exercise and diet routine as planned.

The nurse is providing discharge instructions for a client with Graves disease who has ophthalmopathy. What should the nurse be sure to include in the instructions to decrease exacerbation of this clinical manifestation?

The client should be strongly urged not to smoke. Explanation: Ophthalmopathy can also be aggravated by smoking, which should be strongly discouraged. It is not necessary for the client to avoid contact with others. Alcohol is not contraindicated but should be limited when taking any medication regimen. The client should not adjust the doses of medications without first consulting the physician.

A client undergoing an evaluation of hormone levels asks, "What regulates the hormone levels?" Which response by the nurse would be considered most accurate?

The hypothalamic-pituitary-target cell system Explanation: The levels of many of the hormones are regulated by feedback mechanisms that involve the hypothalamic-pituitary-target cell system. Positive feedback control refers to rising levels of a hormone that causes another gland to release a hormone that is stimulating to the first. The hypophyseal portal system connects the supraoptic and paraventricular nuclei of the hypothalamus with the posterior pituitary gland. Exogenous forms of hormones (given as drug preparations) can influence the normal feedback control of hormone production and release.

The nurse is discussing the administration of GH with a parent of a child who has short stature caused by growth hormone (GH) deficiency. What should the nurse include when educating the parents about administration?

The medication will be given daily during the period of active growth and can be continued into adulthood. Explanation: GH is administered by daily subcutaneous injection during the period of active growth, and can be continued into adulthood.

The nurse is teaching a newly diagnosed client with diabetes about fingerstick blood glucose testing. When the client obtains a premeal fingerstick reading of 206 mg/dL (11.43 mmol/L), which teaching by the nurse is most appropriate?

This is an elevated reading - let's talk about adherence to insulin and diet. Explanation: Normal fasting or pre-meal blood glucose levels are between 70 and 110 mg/dL (3.9 to 6.1 mmol/L). The nurse should ensure the client understands the diet and medication regimen, as diabetes is a self-managed disease; maintaining good glucose control can prevent complications of the disease.

A client who has just undergone a thyroidectomy is experiencing high fever, tachycardia, and extreme restlessness. The nurse would interpret these findings as manifestations of which complication?

Thyroid storm Explanation: The symptoms this client is experiencing are related to thyroid storm and must be treated immediately to prevent death. Myxedema coma is related to hypothyroidism but typically does not occur after a thyroidectomy. Addisonian crisis is related to hypoadrenalism.

For which endocrine disorders would autoimmune antibody testing be appropriate? Select all that apply.

Type 1 diabetes mellitus Graves disease Hypoparathyroidism Addison disease Explanation: Autoantibody testing is most commonly used for autoimmune disorders such as Hashimoto thyroiditis, type 1 diabetes mellitus, Graves disease, hypoparathyroidism, and Addison disease.

A client with type 1 diabetes mellitus wishes to stop taking insulin injections. What option is appropriate?

Using an insulin infusion pump Explanation: Clients with type 1 diabetes mellitus require exogenous insulin because they have absolute lack of their own secretion. Insulin is a protein that would be digested if taken orally. To avoid injections, clients could use an insulin pump, which provides continuous infusion through a catheter placed under the skin that is changed every few days. While some herbal preparations may help lower blood glucose, they do not replace insulin. While aerobic activity helps mobilize glucose, it does not replace insulin. Metformin is an insulin sensitizer that enhances insulin utilization, but does not replace insulin.

Both prehepatic and post-hepatic causes of portal hypertension include the formation of which abnormality?

Venous thrombosis Explanation: Portal hypertension can be caused by a variety of conditions that increase resistance to hepatic blood flow. Prehepatic causes of portal hypertension include portal vein thrombosis; post-hepatic obstruction is caused by conditions such as hepatic vein thrombosis and veno-occlusive disease. Fibrous tissue bands and fibrous nodules that increase the resistance to portal blood flow are intrahepatic causes of portal hypertension. With the gradual obstruction of venous blood flow in the liver, portal vein pressure increases, resulting in the development of collateral channels between the portal and systemic veins. The increased pressure also leads to the formation of portopulmonary shunts that cause blood to bypass the pulmonary capillaries, thus interfering with blood oxygenation and producing cyanosis.

Biopsy results reveal that a client has a deficient amount of parietal (oxyntic) cells in his stomach. The client asks the nurse to explain what this means. The nurse explains that parietal cells secrete HCl and intrinsic factor, which is needed for absorption of:

Vitamin B12 Explanation: The oxyntic glands are located on the inside surfaces of the body and fundus of the stomach. They secrete HCl and intrinsic factor, which is needed for vitamin B12 absorption.

The nurse is caring for a client with diabetes who has developed gastroparesis. Which symptom does the nurse expect the client to report?

Vomiting after eating Explanation: Gastroparesis (delayed emptying of stomach) is characterized by reports of epigastric discomfort, nausea, postprandial vomiting, bloating, and early satiety.

Rotavirus is most severe in children under 24 months of age. What is a typical symptom of rotavirus infection?

Vomiting that disappears around the second day but diarrhea continues Explanation: Rotavirus infection typically begins after an incubation period of less than 24 hours, with mild to moderate fever, and vomiting, followed by onset of frequent watery, stools. The fever and vomiting usually disappear on about the second day, but the diarrhea continues for 5 to 7 days. Dehydration may develop rapidly, particularly in infants.

An obese adult has recently been diagnosed with type 2 diabetes. The nurse knows that the most likely treatment plan for this client will include which topics?

Weight loss, glucose monitoring, and oral antihyperglycemic medications Explanation: Persons with type 2 diabetes would be unlikely to require insulin initially and oral medications are likely to be of benefit as an addition to weight loss and glucose monitoring.

A nurse providing dietary guidance to a client with celiac disease tells the client that which food should be avoided?

Wheat bread Explanation: Clients with celiac disease should avoid ingesting gluten, which is the primary protein in wheat, barley, and rye. The nurse should tell the client to avoid wheat bread. Watermelon, homogenized milk, and pork products do not contain gluten and therefore do not need to be avoided by a client with celiac disease.

While reviewing the colonic absorption and role of flora in the GI system, the instructor will stress that the large intestine contains:

a complex microbial system that contains hundreds of different species. Explanation: The stomach and small intestine contain only a few species of bacteria, probably because the composition of luminal contents (i.e., acids, bile, pancreatic secretions) kills most ingested microorganisms, and the propulsive movements of these organs impedes their colonization. The large intestine, on the other hand, contains a large and complex microbial ecosystem. It has been estimated that each individual has 300 to 500 different species of intestinal bacteria, with anaerobic bacteria outnumbering aerobic bacteria by a large percentage.

When explaining acute pancreatitis to a newly diagnosed client, the nurse will emphasize that the pathogenesis begins with an inflammatory process whereby:

activated pancreatic enzymes escape into surrounding tissues, causing autodigestion of pancreatic tissue. Explanation: Acute pancreatitis is associated with the escape of activated pancreatic enzymes into the pancreas and surrounding tissues. These enzymes cause fat necrosis, or autodigestion, of the pancreas. Alcohol is known to be a potent stimulator of pancreatic secretions, and it also is known to cause partial obstruction of the sphincter of the pancreatic duct, rather than bowel obstruction. The pancreas is irreversibly damaged and will not recover to normal functioning results from chronic pancreatitis. Acute pancreatitis also is associated with viral infections. The pancreas will hypertrophy (enlarge) to the point of causing bowel obstruction occurring with cancer of the pancreas. Presence of stones in the common bile duct with jaundice is primarily a result of gallstones.

A client with type 1 diabetes has started a new exercise routine. Knowing there may be some increased risks associated with exercise, the health care provider should encourage the client to:

carry a snack with a rapidly absorbed form of glucose to prevent profound hypoglycemia. Explanation: People with diabetes are usually aware that delayed hypoglycemia can occur after exercise. Although muscle uptake of glucose increases significantly, the ability to maintain blood glucose levels is hampered by failure to suppress the absorption of injected insulin and activate the counterregulatory mechanisms that maintain blood glucose (to cause a hyperglycemia response). Even after exercise ceases, insulin's lowering effect on blood glucose levels continues, resulting in profound symptomatic hypoglycemia. Treatment of hypoglycemia involves administration of a rapidly absorbed form of glucose. Rapid weight loss accompanies the polyuria and dehydration of hyperglycemia rather than hypoglycemia. Respiratory disorders are associated with preexisting pulmonary or vascular problems exacerbated by the period of exercise.

The nurse is reviewing the medical history of four clients. Which client is at highest risk for developing peptic ulcer disease?

client with a prior diagnosis of Helicobacter pylori who refused treatment Explanation: The two most important risk factors for peptic ulcer disease (PUD) are infection with the bacteria H. pylori and use of aspirin and/or nonsterioidal anti-inflammatory drugs (NSAIDs). Acetaminophen is not an NSAID, so it does not place the client at risk increased risk for PUD. A history of a ruptured appendix will not cause PUD. A high fat intake can be linked to increased risk for gastritis, but a diet high in carbohydrates is not linked to increased risk for PUD.

A client experiences an increase in thyroid hormone as a result of a thyroid tumor. Which hormonal response demonstrates the negative feedback mechanism?

decreased thyroid-stimulating hormone (TSH) Explanation: Negative feedback occurs when secretion of one hormone causes a reduction in the secretion of the hormone that stimulates production of the first hormone. In this case, TSH, which is manufactured by the anterior pituitary gland, would normally stimulate release of thyroid hormones. When there is an excess of thyroid hormones present, there should be a reduction in TSH levels (i.e., there is no need to stimulate more thyroid hormone production when there is already an excess present). The other hormones listed do not have a negative feedback relationship with thyroid hormone production.

The nurse is caring for a client with advanced liver disease who has ascites. Which treatment will the nurse anticipate being used for the daily management of ascites?

diuretics Explanation: The daily management of ascites usually focuses on sodium restriction and administration of diuretics to prevent fluid retention. Loop or potassium-sparing diuretics are used. Large-volume paracentesis is reserved for people with massive ascites that is affecting client comfort. This invasive procedure carries risks and should be avoided if possible through the daily management of the ascites. A thoracentesis involves draining fluid from the pleural cavity (pleural effusion) not the peritoneal cavity. Desmopressin is vasopressin which causes fluid retention and will result in a decrease urine output, which is contraindicated in the treatment of ascites.

The nurse is caring for a client with a hormonal disorder that is widely affecting many body systems. The nurse recognizes that which form of hormone signalling pathway is most likely involved in the client's disorder?

endocrine Explanation: The endocrine pathway involves hormones being produced, secreted, and traveling through blood vessels to distant cells to have an effect, thus having the potential to affect entire body systems. In the paracrine pathway hormones are produced in a cell, secreted, and act directly on nearby receptive cells. The autocrine pathway is similar to the paracrine pathway except that the receptor cells are also secretory cells meaning the cell is able to produce the hormone and exert an effect on itself. There is no pathway known as local mediation.

The endocrine system is closely linked with the nervous system. Which neurotransmitter can also act as a hormone?

epinephrine Explanation: Neurotransmitters such as epinephrine can act as neurotransmitters or as hormones and are referred to as neurohormones. Other examples include dopamine, oxytocin, and somatostatin. Thyroid stimulating hormone, progesterone, and norepinephrine do not act as neurotransmitters.

A client with chronic pancreatitis is experiencing an episode of recurring epigastric pain. The nurse teaches the client to limit which type of meal to avoid further episodes?

fried chicken and french fries Explanation: Of the meal options, fried chicken and french fries should be avoided since a low-fat diet is usually prescribed for clients with chronic pancreatitis.

Caregivers of a toddler report poor oral intake and grimacing when swallowing. The caregivers also describe the toddler as having difficulty with sleep and wheezing being heard when breathing. The nurse focuses initial assessments based on which likely cause of the symptoms?

gastrointestinal reflux disease Explanation: Esophagitis secondary to reflux can cause feeding problems, early satiety, and sleep interruption. Respiratory symptoms may occur due to damage to the respiratory mucosa when gastric reflux enters the esophagus. Asthma may co-occur with gastrointestinal reflux disease in about 50% of children. Rotavirus causes diarrhea and vomiting but not the other symptoms. Irritable bowel syndrome does not cause esophagitis. Peptic ulcer disease would not cause difficulty swallowing.

Which physiologic process best exemplifies a positive feedback mechanism?

increase in prolactin secretion that occurs with more frequent breastfeeding Explanation: A positive feedback mechanism occurs when one hormonal or physiologic factor stimulates further hormonal release, creating a cascade that will increase until the stimulus is reduced. Because the stimulation of breastfeeding releases prolactin that enhances milk production, this is a positive feedback mechanism. Correction of alterations in homeostasis is normally achieved using negative feedback mechanisms, such as those accomplished by insulin, parathyroid hormone, and antidiuretic hormone. In each of these cases, the stimulus results in an response that causes an opposite physiologic effect. For example, high blood glucose levels stimulate the release of insulin which then results in a lowering of the blood glucose level.

The nurse assesses a client in the emergency department with reports of abdominal pain. Which assessment finding will the nurse interpret as supporting appendicitis?

increased pain when pressure to the lower right quandrant is released Explanation: Appendicitis usually has an abrupt onset, with pain referred to the epigastric or periumbilical area that becomes more localized to the right lower quadrant over 2 to 12 hours. The nurse will interpret rebound tenderness, which is pain that occurs when pressure is applied to the area and then released, as supporting appendicitis. Although nausea can accompany the pain, it should not be intermittent over several days as the symptoms progress quickly. Palpation of the abdomen usually reveals a deep tenderness in the lower right quadrant, which is confined to a small area. Urination should not alter the pain and appendicitis pain does not typically occur the upper quadrants.

A client has been diagnosed with alcohol-induced liver disease. He admits to the nurse, "I know what the lungs do, and I know what the heart does, but honestly, I have no idea what the liver does in the body." The nurse should tell the client that the liver:

metabolizes most components of food and also cleans the blood of bacteria and drugs. Explanation: Protein, carbohydrate, and fat metabolism are performed by the liver. As well, it metabolizes drugs and removes bacteria by Kupffer cells. Absorption of nutrients takes place in the intestines. The liver does not produce the bulk of fluids secreted in the GI tract. The liver does not have a primary role in the maintenance of acid-base or electrolyte balance.

A client with severe hypoglycemia is unconscious. Which method of providing glucose should not be used for this client?

orange juice orally Explanation: When clients are unconscious it is not safe to attempt to have them swallow liquids. Alternate routes that reduce the risk of choking such as buccal absorption, intramuscular or intravenous injections are preferred.

An older adult client presents with a perforation of a peptic ulcer. The nurse will monitor for signs and symptoms of which priority complication?

peritonitis Explanation: Perforation occurs when an ulcer erodes through all the layers of the stomach or duodenum wall. With perforation, gastrointestinal contents enter the peritoneum and cause peritonitis. Although the client may experience vomiting or diarrhea, these are not the priority compared to peritonitis.

The nurse is teaching a client newly diagnosed with hypothyroidism. The nurse teaches that thyroid hormone is transported by which?

protein carriers Explanation: Some hormones, such as steroids and thyroid hormone, are bound to protein carriers for transportation to the target cell destination. The extent of carrier binding influences the rate at which hormones leave the blood and enter the cells. Cholesterol is a precursor for steroid hormone. Prohormones have an extra amino acid and are converted to hormones in the Golgi complex.

During a lecture about the function of the intestine related to food digestion, the faculty mentions that when the students consume foods high in acid, the intestines will:

stimulate the release of secretin, which then inhibits release of gastrin. Explanation: Secretin, which is secreted by S cells in the mucosa of the duodenum and jejunum, inhibits gastric acid secretion. The entry of an acid chyme into the intestine stimulates the release of secretin, which inhibits the release of gastrin. Several gut-derived hormones have been identified as having what is termed an incretin effect, meaning that they increase insulin release after an oral glucose load. This suggests that gut-derived factors can stimulate insulin secretion after a high-carbohydrate meal. The two hormones that account for about 90% of the incretin effect are glucagon-like peptide 1 (GLP-1) and glucose-dependent insulinotropic peptide (GIP). Because increased levels of GLP-1 and GIP can lower blood glucose levels by augmenting insulin release in a glucose-dependent manner (i.e., at low blood glucose levels) no further insulin is secreted, minimizing the risk of hypoglycemia.

In major athletic competition, athletes are required to submit to liquid chromatography testing looking for:

use of performance-enhancing agents to increase the chances of winning. Explanation: For some steroid or peptide hormones, mass spectrometry is becoming increasingly useful and can be combined with other analytical techniques, such as liquid chromatography. These approaches provide definitive identification of the relevant hormone or compound according to its chemical or physical characteristics (e.g., unequivocal detection of performance-enhancing agents in sports).

A client is scheduled for a suppression test as part of the diagnostic testing for his suspected endocrine disorder. The results of this test will help the care team determine:

whether the client is producing excessive hormone levels. Explanation: Suppression tests are used when hyperfunction of an endocrine organ is suspected. Suppression tests are not used to gauge pituitary or hypothalamus function. Stimulation tests are used to rule out hypofunction of an endocrine organ.


संबंधित स्टडी सेट्स

M03-Ch.14&Ch.15- Troubleshooting Windows startup

View Set

Level I Antiterrorism Training - Pre Test

View Set

Chapter 22: Genomics I: Analysis of DNA

View Set

Jewish Feasts/Festivals Unit 2: Bible 7

View Set

Skills and Essentials Ch 28 Infection Control

View Set

2 blood types, blood transfusion and tissue and organ transplantation

View Set